Google Groups no longer supports new Usenet posts or subscriptions. Historical content remains viewable.
Dismiss

Poincaré über transfinite Zahlen

559 views
Skip to first unread message

JVR

unread,
Feb 5, 2024, 11:57:58 AMFeb 5
to
Es gibt einen Vortrag von Poincaré, gehalten in Göttingen im Jahr 1909,
worin der scheinbare Widerspruch zwischen dem Beweis, dass die reellen
Zahlen nicht abzählbar sind, und der Aussage von Richard, dass alle
definierbaren Begriffe abzählbar sein müssen, aufgelöst wird.

Poincaré beweist, wie Cantor es schon getan hat, dass die Überabzählbarkeit
des Kontinuums aus der topologischen Vollständigkeit folgt.

Kurioserweise wird Poincaré aber gerne von anti-Cantor-Cranks zitiert.

Ganzhinterseher

unread,
Feb 6, 2024, 3:19:05 AMFeb 6
to JVR
Schön, dass Du inzwischen auch auf diese Arbeit gestoßen bist.
Cantor und Richard widersprechen einander.
Mit einem Anflug von Matheologie behauptet Poincaré, sie sind beide richtig.
Ich behaupte, sie sind beide falsch. Und ich kann es beweisen.

Es gibt keine Abzählung unendlicher Mengen. Das einfachste Beispiel ist
die Folge der Endsegmente. Jedes ist unendlich, definiert also unendlich
viele natürliche Zahlen *nicht*.
Ein ebenso einfaches Beispiel ist die Folge der Stammbrüche. Zu jedem
eps > 0 gibt es unendlich viele kleinere, die also durch dieses eps
nicht definierbar sind und damit durch kein eps definierbar sind. Und
schließlich der gescheiterte Versuch, alle Brüche mit Ganzzahlbrüchen zu
indizieren, d.h. die O in der Matrix
XOOO...
XOOO...
XOOO...
XOOO...
...
durch unendlich viele endlich indizierte Umordnungen zum Verschwinden zu
bringen. Jede definierbare Umordnung erreicht genau gar nichts.

Übrigens sagt Cantor selbst: "Wäre Königs Satz, daß alle 'endlich
definirbaren' reellen Zahlen einen Inbegriff von der Mächtigkeit ℵo
ausmachen, richtig, so hieße dies, das ganze Zahlencontinuum sei
abzählbar, was doch sicherlich falsch ist." Natürlich, alles dies ist
falsch. Aber Cantor hat jedenfalls recht, wenn er auf aktualer
Unendlichkeit als Grundlage seiner Theorie besteht. Andernfalls wäre
alles von vornherein als Humbug erkenntlich. Und da Poincaré am Ende
seines Vortrags behauptet; "Ein aktuales Unendliche gibt es jedenfalls
nicht." hätte er sich das Ganze auch sparen können.

Natürlich gibt es unter der Voraussetzung aktualer Unendlichkeit mehr
reelle Zahlen als rationale Zahlen, denn jede rationale Zahl ist reell,
aber nicht jede relle Zahl ist rational. Ebenso gibt es mehr rationale
Zahlen als ganze, denn jede ganze Zahl ist rational, aber nicht jede
rationale Zahl ist ganz. Das hat aber nichts mit Cantors
Abzählungsversuchen zu tun.

Gruß, WM

JVR

unread,
Feb 6, 2024, 6:34:36 AMFeb 6
to
Der Beweis, dass ein topologisches Kontinuum (d.h. ein zusammenhängender
kompakter Hausdorff-Raum - ja, ich weiß, Sie haben keine Ahnung was das
sein könnte) nicht abzählbar ist, den Poincaré in dem zitierten Aufsatz bringt,
stammt von Cantor und ist klar und einfach und widerspricht Ihrer Kraut-und-
Rüben-Logik.

Nebenbei bemerkt: Cantor beweist auch, dass schon eine nirgends dichte
perfekte Menge überabzählbar sein muss. Wie sind eigentlich Begriffe wie
'dicht', 'nirgends dicht', 'abgeschlossen', 'offen', 'perfekt' in der neuen
mückmeatischen Mathematik definiert?

Sie palavern fröhlich über 'das aktual Unendliche' und 'das potentiell Unendliche',
ohne diese Begriffe definieren zu können.

Sie können offensichtlich weder genügend genau lesen, noch schreiben,
um Mathematik zu verstehen oder auch nur einfache logische
Zusammenhänge zu formulieren.

Jens Kallup

unread,
Feb 6, 2024, 6:55:17 AMFeb 6
to
Am 2024-02-06 um 12:34 schrieb JVR:
> Sie palavern fröhlich über 'das aktual Unendliche' und 'das potentiell Unendliche',
> ohne diese Begriffe definieren zu können.

wie war das:
- aktuale oder potentielle oo - eines von den beiden hat doch Cantor
so beschrieben, das eine der beiden nur "erdacht" wurde, um die andere
zu beschreiben... ?

- was dann als (in meiner Sicht) Lemma dient - kann man ja machen, im
Deutsch-, Physik-, und Mathe-buch/heft ein paar Notizen.

- selbst Goethe hat einen Strich in seinen Tagebuch gemacht, einer für
jede "wohlustige" Begebenheit - so gegen 22 Uhr...

Jens

--
Diese E-Mail wurde von Avast-Antivirussoftware auf Viren geprüft.
www.avast.com

Ganzhinterseher

unread,
Feb 6, 2024, 9:43:06 AMFeb 6
to JVR
On 06.02.2024 12:34, JVR wrote:
> On Tuesday, February 6, 2024 at 9:19:05 AM UTC+1, Ganzhinterseher wrote:
>> Schön, dass Du inzwischen auch auf diese Arbeit gestoßen bist.

Dir ist wohl noch so manches neu und unverständlich in diesem Themenakreis.

> Sie palavern fröhlich über 'das aktual Unendliche' und 'das potentiell Unendliche',
> ohne diese Begriffe definieren zu können.

Weshalb sollte ich sie definieren? Cantor und Poincaré haben sie
definiert und benutzen sie. Man versteht sie, oder man versteht sie
nicht. Dass Du sie nicht verstehen kannst, ist inzwischen kein Geheimnis
mehr. Aber warum sollte ich mich um Deine Erkenntnislücken kümmern?
>
> Sie können offensichtlich weder genügend genau lesen, noch schreiben,
> um Mathematik zu verstehen oder auch nur einfache logische
> Zusammenhänge zu formulieren.

Willst Du den Rosenthal 2 machen? Hast Du Poincarés Artikel gelesen?
"Ein aktuales Unendliche gibt es jedenfalls nicht." Was hast Du Dir
dabei eigentlich gedacht?

Aber ich hatte doch drei Punkte genannt, die Du verstehen solltest.
Denke darüber nach, anstatt nur Gift zu speien.

Gruß, WM

JVR

unread,
Feb 6, 2024, 10:21:14 AMFeb 6
to
Das Wort 'aktual' kommt in Poincarés Aufsatz genau einmal vor. Was
mit der Behauptung "Ein aktual Unendliches gibt es jedenfalls nicht"
gemeint ist, geht daher aus dem Zusammenhang nicht hervor. Der
Zusammenhang ist der Ausdruck folgenden Zweifels:

"Was nun die zweite transfinite Kardinalzahl Aleph_1 betrifft, so bin ich nicht
ganz überzeugt, daß sie existiert. Man gelangt zu ihr durch Betrachtung der
Gesamtheit der Ordnungszahlen von der Mächtigkeit Aleph_0; es ist klar, daß diese
Gesamtheit von höherer Mächtigkeit sein muß. Es fragt sich aber, ob sie abgeschlossen
ist, ob wir also von ihrer Mächtigkeit ohne Widerspruch sprechen
dürfen."

Die Ursache des Zweifels ist ganz wesentlich subtiler als die mückmeatischen
Überlegungen und bezieht sich auf die 'Definierbarkeit' der höheren Alephs .

Es gibt einen interessanten Briefwechsel zwischen Borel, Lebesgue, Baire und
Hadamard zur Frage, wie Zermelos Satz zu verstehen ist. Poincaré nennt den
Satz 'fast genial'. Aber es herrscht Uneinigkeit über dessen Bedeutung. Der
Zweifel besteht heute noch, hat aber mit dem primitiven Konzept der
undefinierten 'potentiellen Unendlichkeit' nichts zu tun.

WM

unread,
Feb 6, 2024, 11:09:35 AMFeb 6
to
> Das Wort 'aktual' kommt in Poincarés Aufsatz genau einmal vor.

An anderer Stellen benutzt er es öfter. Hier eine kleine Auswahl seiner
Gedanken zum Cantorism:

For the greater number of us these prejudices had been dissipated, but
it has come about that we have run against certain paradoxes and
apparent contradictions, which would have rejoiced the heart of Zeno of
Elea and the school of Megara. Then began the business of searching for
a remedy, each man his own way. For my part I think, and I am not alone
in so thinking, that the important thing is never to introduce any
entities but such as can be completely defined in a finite number of
words. Whatever be the remedy adopted, we can promise ourselves the joy
of the doctor called in to follow a fine pathological case. [H.
Poincaré: "Science and method: The future of mathematics", Nelson,
London (1914) p. 44f]

There is no actual infinity. The Cantorians forgot this, and so fell
into contradiction. [H. Poincaré: "Science and method: Last efforts of
logisticians", Nelson, London (1914) p. 195]

1. Never consider other objects than those which can be defined by a
finite number of words. 2. Never forget that every proposition about the
infinite is only a substitution, an abbreviated expression of a
proposition about the finite. [H. Poincaré: "Letzte Gedanken: Die Logik
des Unendlichen", übers. von K. Lichtenecker, Akad. Verlagsgesellschaft,
Leipzig (1913) p. 141f]

> Was
> mit der Behauptung "Ein aktual Unendliches gibt es jedenfalls nicht"
> gemeint ist, geht daher aus dem Zusammenhang nicht hervor.

Für jeden Leser mit mathematischer Grundausbildung ist es klar.
>
> Es gibt einen interessanten Briefwechsel zwischen Borel, Lebesgue,
Baire und
> Hadamard zur Frage, wie Zermelos Satz zu verstehen ist. Poincaré
nennt den
> Satz 'fast genial'. Aber es herrscht Uneinigkeit über dessen
Bedeutung. Der
> Zweifel besteht heute noch, hat aber mit dem primitiven Konzept der
> undefinierten 'potentiellen Unendlichkeit' nichts zu tun.

Du hast zwar keine Ahnung, um was es sich handelt, aber Du weißt, dass
es ein primitives Konzept ist.

Zu meinen drei Argumenten hast Du aber nichts zu sagen? Verstehst Du sie
auch nicht?

Gruß, WM

JVR

unread,
Feb 6, 2024, 11:31:52 AMFeb 6
to
Richtig - zu Ihrer primitiven Polemik habe ich nichts zu sagen.

„Es ist schon alles gesagt, nur noch nicht von allen.“
-- Karl Valentin

Martin Vaeth

unread,
Feb 6, 2024, 3:24:07 PMFeb 6
to
JVR <jrenne...@googlemail.com> schrieb:
>
> "Was nun die zweite transfinite Kardinalzahl Aleph_1 betrifft,
> so bin ich nicht ganz überzeugt, daß sie existiert.
> Man gelangt zu ihr durch Betrachtung der Gesamtheit der
> Ordnungszahlen von der Mächtigkeit Aleph_0; es ist klar, daß
> diese Gesamtheit von höherer Mächtigkeit sein muß. Es fragt
> sich aber, ob sie abgeschlossen ist, ob wir also von ihrer
> Mächtigkeit ohne Widerspruch sprechen dürfen."

Da freue ich mich doch, dass ich diese Gruppe noch ab und
zu lese: Das ist ja eine echte Perle.

Poincare hat also i.W. Zweifel am Ersetzungsaxiom
(und ist damit nicht allein, s. unten):

Dieses wurde ja i.W. *nur* dazu eingeführt, um axiomatisch
den Beweis des Satzes von Hartogs zu ermöglichen.
In der Tat ist beispielsweisew die Menge aller Wohlordnungen
auf aleph_0 natürlich eine *Menge*, und nach dem Ersetzungsaxiom
muss daher das funktoriell zugeordnete System aller abzählbaren
Ordinalzahlen ebenfalls eine Menge bilden (was wohl das ist,
was Poincare im Text oben als "abgeschlossen" bezeichnet).

Tatsächlich gibt es große Mengentheoretiker, die
Zweifel an der Konsistenz von Z mit dem Ersetzungsaxiom
haben: Das Ersetzungsaxiom ist wirklich alles andere als
intuitiv richtig. Es gibt wohl mehr als einen, der schon
versucht hat, damit eine Inkonsistenz zu finden.

(Leider erinnere ich mich nur dunkel, wo ich das gelesen
habe; vermutlich in mathoverflow, und einer der Zweifler,
die lange einen Widerspruch suchten, ist WIMRE jemand, der
mit dem großen Solovay publiziert hat.)

Allerdings scheinen sich die großen Mengentheoretiker
einig zu sein, dass man die Konsistenz beibehält, wenn
man das Ersetzungsaxiom für eine gewisse Klasse von
Funktoren einschränkt, die immer noch groß genug ist,
um den Satz von Hartogs zu beweisen. Aber trotzdem
zeigt diese Diskussion großer Geister m.E., dass
Poincares Kritik in der Tat berechtigt ist: Nur, weil
etwas möglicherweise konsistent ist, muss es noch
lange nicht eine gute Wahl für ein Axiom sein.

Martin Vaeth

unread,
Feb 6, 2024, 4:09:03 PMFeb 6
to
Martin Vaeth <mar...@mvath.de> schrieb:
>
> (Leider erinnere ich mich nur dunkel, wo ich das gelesen
> habe; vermutlich in mathoverflow, und einer der Zweifler,
> die lange einen Widerspruch suchten, ist WIMRE jemand, der
> mit dem großen Solovay publiziert hat.)

Fast: mathoverflow und Solovay waren zumindest die richtigen
Stichworte: https://mathoverflow.net/questions/260944/

Martin Vaeth

unread,
Feb 6, 2024, 4:30:52 PMFeb 6
to
Genauer meinte ich den Verweis auf den Link aus dem
Kommentar von Timothy Chow auf die obige Frage, der
ebenfalls ein Posting von Solovay enthält:
https://cs.nyu.edu/pipermail/fom/2007-August/011835.html

Dort steht auch, dass Randall Holmes meint, dass
die Abschwächung des Ersetzungsaxioms auf \Sigma_2-Funktoren
(sind das solche, deren Formalisierung nur die Quantorreihenfolge
\forall\exists\forall oder \exists\forall\exists erlauben?
Das sollte doch für Hartogs ausreichen, oder?)
weniger Gefahr läuft, inkonsistent zu sein.

JVR

unread,
Feb 6, 2024, 5:41:17 PMFeb 6
to
In der Tat ist es so, dass es für Amateure und Dilettanten völlig sinnlos ist,
sich als Vertreter der einen oder anderen Position zu gerieren.
Oder glaubt jemand hier, er sei in der Lage, die Unterschiede in der
Auffassung von Hadamard und Borel überzeugend auszuräumen?

WM

unread,
Feb 7, 2024, 12:34:14 PMFeb 7
to
Martin Vaeth schrieb am Dienstag, 6. Februar 2024 um 22:30:52 UTC+1:

> Genauer meinte ich den Verweis auf den Link aus dem
> Kommentar von Timothy Chow

Wer Chow zitiert, offenbart ein niedriges Niveau. Denn er hat Chow nicht durchchowt. Der tut so, als hätte er Argumente, aber bei näherer Untersuchung bringt er nur Behauptungen, wie hier:
"A careful study of ZFC necessarily requires keeping track of exactly which concepts are absolute and which are not. [...] the majority of basic concepts are absolute, except for those associated with taking powersets and cardinalities," [Timothy Y. Chow: "A beginner's guide to forcing", arXiv (2008) p. 6f] Eine Begründung für diese Ausnahmen wird nirgends gegeben.

Eine genauere Analyse der ZFC-Mengenlehre ist außerdem sinnlos, denn Zermelo "beweist" seinen Wohlordnungssatz nur für abzählbare Mengen.

"Because the first element of every -set is m1 since the corresponding segment A does not contain an element, i.e., M - A = M. If now m' were the first element of M' which differed from the corresponding element m'' then the corresponding segments A' and A'' must be equal and hence also the complementary sets M - A' and M - A'' and as their distinguished elements m' and m'' themselves, contrary to the assumption." [E. Zermelo: "Beweis, daß jede Menge wohlgeordnet werden kann", Math. Ann. 59 (1904) p. 515]

Here (the first element of M') Zermelo uses a step-by-step argument which is not allowed, because it would presuppose a well-order, the countability and even finiteness of the real numbers:

Denn genau dasselb Argument führt zum Theorem All initial segments C of the well-ordered set |R are countable.

Siehe https://www.hs-augsburg.de/~mueckenh/Transfinity/Transfinity/pdf p. 307.

Gruß, WM

WM

unread,
Feb 7, 2024, 12:42:37 PMFeb 7
to
JVR schrieb am Dienstag, 6. Februar 2024 um 23:41:17 UTC+1:

> In der Tat ist es so, dass es für Amateure und Dilettanten völlig sinnlos ist,
> sich als Vertreter der einen oder anderen Position zu gerieren.
> Oder glaubt jemand hier, er sei in der Lage, die Unterschiede in der
> Auffassung von Hadamard und Borel überzeugend auszuräumen?

Jemand, der den Unterschied zwischen potentiell und aktual unendlich nicht kennt, hat natürlich keine Chance.
Die Diskussion ist im Übrigen sinnlos, weil es, wie Borel behauptet, inaccessible numbers gibt, nämlich fast alle reellen Zahlen. Natürlich kann man die nicht wohlordnen. Er hat sogar ein Buch darüber geschrieben: Les nombres inaccessible:
This little book is the result of half a century of reflections on the
principles of mathematical analysis and, in particular, on the definition
of numbers. Some of these reflections have already been sketched
here and there in the works of this Collection, but it seemed to me
that it would be useful to coordinate them in a connected account.

Borel's thesis is that the overwhelming majority of numbers will
always remain inaccessible to the human race as we know it, in the
sense that it will never be possible to define these numbers effectively
in such a manner that any two mathematicians will be certain that
they are speaking about one and the same entity.

Aber da Du das alles nicht verstehen kannst, ist es natürlich nur billige Polemik.

Gruß, WM

Fritz Feldhase

unread,
Feb 7, 2024, 1:18:36 PMFeb 7
to
On Wednesday, February 7, 2024 at 6:34:14 PM UTC+1, WM wrote:

> Eine genauere Analyse der ZFC-Mengenlehre ist außerdem sinnlos, denn Zermelo "beweist" seinen Wohlordnungssatz nur für abzählbare Mengen.

Mückenheim, Du redest, wie in letzter Zeit praktisch immer, wieder einmal wirres Zeug daher.

Zermelo hat den Wohlordnungssatz im Kontext der "Zermelo-Mengenlehre" bewiesen. Der Beweis kann leicht in ZFC nachvollzogen werden. (Siehe P. R. Halmos: Naive Mengenlehre.)

Hinweis: "Ernst Zermelo führte das Auswahlaxiom als „unbedenkliches logisches Prinzip“ ein, um den Wohlordnungssatz zu beweisen." (Wikipedia)

Siehe: https://de.wikipedia.org/wiki/Zermelo-Mengenlehre
bzw. https://en.wikipedia.org/wiki/Zermelo_set_theory
sowie https://de.wikipedia.org/wiki/Wohlordnungssatz

JVR

unread,
Feb 7, 2024, 3:58:18 PMFeb 7
to
Genauer: Ein billiges argumentum ad hominem.

Jens Kallup

unread,
Feb 8, 2024, 6:13:22 AMFeb 8
to
Am 2024-02-07 um 18:42 schrieb WM:
> they are speaking about one and the same entity.

"... sie werden für gleich gehalten."

> Aber da Du das alles nicht verstehen kannst, ist es natürlich nur billige Polemik.

jojo.
eins ist eins.
eins plus eins ist drei.

oo_0 UND oo_1 ergeben 2.

schon klar, ALLES...

WM

unread,
Feb 8, 2024, 10:25:36 AMFeb 8
to
Fritz Feldhase schrieb am Mittwoch, 7. Februar 2024 um 19:18:36 UTC+1:
> On Wednesday, February 7, 2024 at 6:34:14 PM UTC+1, WM wrote:
>
> > Eine genauere Analyse der ZFC-Mengenlehre ist außerdem sinnlos, denn Zermelo "beweist" seinen Wohlordnungssatz nur für abzählbare Mengen.

> Zermelo hat den Wohlordnungssatz im Kontext der "Zermelo-Mengenlehre" bewiesen. Der Beweis kann leicht in ZFC nachvollzogen werden. (Siehe P. R. Halmos: Naive Mengenlehre.)

Der Beweis enthält die Bestimmung: If now m' were the first element of M' which differed from the corresponding element m''. Das setzt voraus, dass alle Endsegmente linear durchsucht werden können, auch die stets unendlich vielen darin verbleibenden Elemente. Und natürlich funktioniert das nur für abzählbare Mengen.
>
> Hinweis: "Ernst Zermelo führte das Auswahlaxiom als „unbedenkliches logisches Prinzip“ ein, um den Wohlordnungssatz zu beweisen." (Wikipedia)

Falsch, da die meisten Zahlen inaccessible und damit nicht wählbar sind.

Gruß, WM

JVR

unread,
Feb 8, 2024, 12:39:13 PMFeb 8
to
Wenn man Literatur zitiert, gibt man normalerweise die Quelle an.
Welcher Beweis ist Ihrer Meinung nach fehlerhaft?
Der von 1904 (Annalen Bd. 59., p. 514)
oder der von 1907 (Annalen Bd. 65, p. 107)?

Und warum genau? Was Sie oben schreiben ist keine Begründung.

Jens Kallup

unread,
Feb 8, 2024, 1:01:58 PMFeb 8
to
Am 2024-02-08 um 16:19 schrieb WM:
>> Noch ein kleiner Hinweis, JEDER Rohrzustand enthält unendlich viele
Murmeln.
> Man könnte also mit den freigesetzten Murmeln niemals alle Brüche
indizieren. Willst Du das behaupten?
das ist richtig.

- "Jeder" Moment "einer" Betrachtung "einer" oo, ergibt "einen" Zustand
- "Jeder" weitere Moment ergibt "eine" weitere "eine" Betrachtung

- hat man einen Quantor, so kann es möglich sein, das man "zwei" Momente
oder mehr hat !
weil: ein Quantor besagt, "das mindestens eine Eigenschaft existiert -
es können aber auch mehrere, oder auch oo viele sein..."
- wenn man aber bei oo Symbolen (ob nun aleph, epsilon, omega...) ist,
dann ist das Ergebnis "nicht bestimmt" bzw. "nicht bestimmbar", weil:
WIR WISSEN NICHT, WIE GROSS ODER KLEIN ES IST !

weil aber die Objekte als "gedacht" vorhanden sind, so wird "eine"
Gruppe von Objekten (in anderen Bereichen, eine Gruppe von Induvidien)
als eins (1 = vorhanden deklariert) -
weil:
- wenn wir eine Gruppe als (0 = nicht vorhanden/leer) deklarieren
dann kann "eins" (1) nicht kleiner als 0 UND nicht größer als 1 sein
weil:
- im Sprachgebrauch: der Mensch (ein junger Mensch) wächst mit der
Zeit - also er wird größer - aber das steht dann in einen anderen
Kontext als dessen, das der "eine" Mensch IMMER der "gleiche" sein
wird, sich aber durchaus von den anderen unterscheidet.
JEDER Mensch ist daher "einzig"Artik !
das "Leben" ist "einzig"Artig (und es muss beschützt werden) !!!

=> (daraus folgt:) Das sich "eine" Gruppe oder "ein" Häuptling wie
Wolfgang geben muss, der die Gruppe(n) in der Hand hat, leiten,
und erziehen kann.

=> (daraus folgt:) Das fäkalische Sprache nicht gerade förderlich
sich auf die Entwicklung ausübt.
Jedenfalls im mathematischen Bereich, im biologischen Bereich
sieht das dann natürlich anders aus, wo man dann wirklich sagen
oder schreiben kann: "das ist aber nun über Deinen Mist..."
Nur so nebenbei: Pilze vom Aldi oder Netto, die werden auch auf
Pferdemist gezüchtet - und die Kühe sind auch bei der Firma
Milka "nicht" lila.

- in der Fuzzy-Logik (also schwammigen, aber auch "scharfen" Logik)
wiegt der Wert "eins" (1) "mehr" als "null" (0):
weil:
=> 1 überdeckt ALLE (leeren (iduktive) Mengen) oder ALLE Stammbrüche
=> x > 0. x := 1. x > 0

=> 1 ist dann so gesehen, die größte reelle Zahl, wenn 1/n, 1/n+1
gilt.
=> 1 ist auch die kleinste Zahl,
weil (auf oo Kontext-bezugnehmend) von "einer" kleinsten reellen
Zahl, auch "eine" weitere kleinste gefunden werden kann -

=> da aber nun zu JEDER kleinsten Zahl niemand im Stande ist je
die kleinste anzugeben, ist die kleinste und auch die größte
oo gleich "eins" (1).

=> Was würde Achim von der Maus sagen: "hört sich komisch an, ist
aber so.".

Jens

Ganzhinterseher

unread,
Feb 8, 2024, 2:28:23 PMFeb 8
to JVR
JVR schrieb am Donnerstag, 8. Februar 2024 um 18:39:13 UTC+1:
> On Thursday, February 8, 2024 at 4:25:36 PM UTC+1, WM wrote:

> > > > Eine genauere Analyse der ZFC-Mengenlehre ist außerdem sinnlos,
denn Zermelo "beweist" seinen Wohlordnungssatz nur für abzählbare Mengen.

> > Der Beweis enthält die Bestimmung: If now m' were the first element
of M' which differed from the corresponding element m''. Das setzt
voraus, dass alle Endsegmente linear durchsucht werden können, auch die
stets unendlich vielen darin verbleibenden Elemente. Und natürlich
funktioniert das nur für abzählbare Mengen.

> Wenn man Literatur zitiert, gibt man normalerweise die Quelle an.

Das tat ich im Parallelposting. Aber hier ist die Quelle nochmals: [E.
Zermelo: "Beweis, daß jede Menge wohlgeordnet werden kann", Math. Ann.
59 (1904) p. 515]

> Was Sie oben schreiben ist keine Begründung.

Doch, denn was man auf einen ersten Fehltritt prüfen kann, ist Schritt
für Schritt prüfbar und damit abzählbar.

Gruß, WM


Jens Kallup

unread,
Feb 8, 2024, 2:53:52 PMFeb 8
to
Am 2024-02-08 um 20:28 schrieb Ganzhinterseher:
> Das tat ich im Parallelposting. Aber hier ist die Quelle nochmals: [E.
> Zermelo: "Beweis, daß jede Menge wohlgeordnet werden kann", Math. Ann.
> 59 (1904) p. 515]
>
> > Was Sie oben schreiben ist keine Begründung.
>
> Doch, denn was man auf einen ersten Fehltritt prüfen kann, ist Schritt
> für Schritt prüfbar und damit abzählbar.

letzeres (WM) ist richtig, wie auch ersteres (ZF) ist richtig.

ALLE Mengen können wohlgeordnet sein.

weil: => M1 := { 0, 1, 2, 3, 4 }. ist wohlgeordnet, Mächtigkeit: 5.
weil: => M2 := { 0 } => { } . ist wohlgeordnet, Mächtigkeit: 1.
weil: => M3 := { oo } => { 1 } . ist wohlgeordnet, Mächtigkeit: 1. [1]
weil: => M4 := { a, e, w } . ist wohlgeordnet, Mächtigkeit: 3. [2]

[1] ALLE (zum Beispiel IN) in M3 sind wohlgeordnet, auch wenn die
vorhandene Mächtigkeit "ein"en (1) Symbolwert entspricht.
Mit nur "einen" Blick erkennt man hier, das man ALLE oo IN Objekte
abgezählt hat, und sie mit "eins" (1) benennt/schreibt/possaunt...

[2] ALLE symbole/kardinal-Objekte sind wohlgeordnet, da:
a := aleph vor:
e := epsilon vor:
w := omega kommt.

Hier gilt zum Beispiel the alphabetetische Ordnung, die mit:
a, e, und w festgelegt wurde.

Euer Schreiberling
Message has been deleted

Fritz Feldhase

unread,
Feb 8, 2024, 2:57:14 PMFeb 8
to
On Thursday, February 8, 2024 at 8:28:23 PM UTC+1, Ganzhinterseher wrote:
> JVR schrieb am Donnerstag, 8. Februar 2024 um 18:39:13 UTC+1:
> > On Thursday, February 8, 2024 at 4:25:36 PM UTC+1, WM wrote:
> > >
> > > Der Beweis enthält die Bestimmung: If now m' were the first element
> > > of M' which differed from the corresponding element m''. Das setzt
> > > voraus, dass alle Endsegmente linear durchsucht werden können, auch die
> > > stets unendlich vielen darin verbleibenden Elemente. Und natürlich
> > > funktioniert das nur für abzählbare Mengen.

Offenbar haben alle anderen Mathematiker diesen Fehler bisher übersehen, Mückenheim.

Wann dürfen wir mit einer entsprechendne Publikation rechnen?

Die mathematische Community wäre sicher überaus interessiert an Ihrer Entdeckung!

> > Was Sie oben schreiben ist keine Begründung.

Du siehst, hier tut ein wissenschaftlicher Artikel, wo Du alles ausführlich erklärst, Not!

Auf diese Weise kannst Du derartige Zweifel an Deiner Entdeckung ein für allemal ausräumen! (Es wird Zeit, dass Deinen "Kritikern" einmal so richtig der Mund gestopft wird!)

> Doch, denn was man auf einen ersten Fehltritt prüfen kann, ist Schritt für Schritt prüfbar und damit abzählbar.

Mückenhem, eine solche flapsige Bemerkung ist meiner Meinung nach nicht wirklich ausreichend, um Ihre Endeckung zu begründen.*) Bitte halten Sie mit einer ausführlichen Begründung nicht länger hinter dem Berg. Die Welt wartet auf Ihren Beitrag!

____________________________________________________________

*) Oder meinen Sie allen Ernstes, dass der folgende Text pubikationswürdig/publizierbar wäre?

==============================================================================

__Ein Fehler in Zermelos Beweis von W. Mückenheim__

Zermelos Beweis*) enthält die Bestimmung: "If now m' were the first element of M' which differed from the corresponding element m''. Das setzt voraus, dass alle Endsegmente linear durchsucht werden können, auch die stets unendlich vielen darin verbleibenden Elemente. Und natürlich funktioniert das nur für abzählbare Mengen. Denn was man auf einen ersten Fehltritt prüfen kann, ist Schritt für Schritt prüfbar und damit abzählbar.
______
*) E. Zermelo: "Beweis, daß jede Menge wohlgeordnet werden kann", Math. Ann. 59 (1904) p. 515

==============================================================================

Sie können es ja mal bei einem einschlägigen Journal versuchen (natürlich dann in englischer Sprache).

JVR

unread,
Feb 8, 2024, 3:08:48 PMFeb 8
to
On Thursday, February 8, 2024 at 8:28:23 PM UTC+1, Ganzhinterseher wrote:
Sie beziehen sich also auf den ersten Paragraphen auf Seite 515.
Offenbar haben Sie übersehen, dass die beiden Mengen M'_{gamma} und
M''_{gamma} wohlgeordnet sind, woraus folgt, dass m' bzw m'' als
erstes Element einer wohldefinierten Untermenge existieren muss.

Im allgemeinen kann man annehmen, dass man in einer Arbeit, die
seit mehr als 100 Jahren jedem Mathematiker bekannt ist, keine
dermaßen elementare Fehler finden wird.

Jens Kallup

unread,
Feb 8, 2024, 3:11:05 PMFeb 8
to
Am 2024-02-08 um 20:55 schrieb Fritz Feldhase:
> Fehltritt prüfen kann, ist Schritt für Schritt prüfbar und damit abzählbar.

ich weiß nicht genau...
aber eine Bilanz oder ein privates Kassenbuch, in dem ALLES so
protokolliert wird, das der getätigte Einkauf genau dem entspricht,
was man eigentlich ausgeben "wollte", ist mir irgendwie schleierhaft
und unverständlich.

Welcher Weinwinzer gibt sein Rezept ab, mit wieviel Fruchtsaftgehalt
an Trauben die Mischung seines Produktes marktreif macht ?

Welcher Bächer gibt sein Rezept an, wie denn die lekkeren Brötchen im
Steinofen so schön kross und trotzdem innen weich und "füllig" bleiben ?

Welcher Fleischbearbeiter gibt seine Würzmischungen und dergleichen der
öffentlichen Gemeindeverwaltung ab, so dass die 10 anderen Fleischer das
gleiche Produkt, aber jeder der 10 Fleischer einen anderen, höheren
Preis für die selbe Arbeit verlangen ?

Welcher Bauer kommt mit seinen Korn als erstes drann, wenn 10 Bauern an
3 Mühlen gleichzeitig stehen ?

Ich mein, Gerechtigkeit hin oder her, aber manchmal muss man doch auch
ein Tierzüchter sein... (selbst die Prinzen habe gesungen: Du musst ein
... auf dieser Welt ...".

Und so pingelich wie der Pabst zu sein - ich weiß nicht, ob das so sehr
erstrebenswert ist.

Es ist klar, 100% Leistung bringen geht nicht.
Und wenn, dann wurde der Apparat "Mensch" sehr schnell kaputt gehen -
schneller als jede Maschiene, die je ein Mensch zuvor gesehen hat...

Jens Kallup

unread,
Feb 8, 2024, 3:20:24 PMFeb 8
to
Am 2024-02-08 um 21:08 schrieb JVR:
> Sie beziehen sich also auf den ersten Paragraphen auf Seite 515.
> Offenbar haben Sie übersehen, dass die beiden Mengen M'_{gamma} und
> M''_{gamma} wohlgeordnet sind, woraus folgt, dass m' bzw m'' als
> erstes Element einer wohldefinierten Untermenge existieren muss.

"...das muss" ist hier falsch.
weil:
- man kann einer induktiven "leeren" Menge weitere induktive "leere"
Mengen hinzufügen - es bleibt IMMER eine induktive "leere" Menge.
- man kann einer induktiven "leeren" Menge weitere aleph, epsilon
Mengen hinzufügen, dann ist die induktive "leere" Menge aber keine
induktive "leere" Menge mehr (die sich durch den Symbolwert 0
definiert), sondern eine "aleph" bzw. "epsilon" Menge wird, die sich
mit den Symbolwert 1 definiert.
- wenn man also mehrere Symbol-Mengen zusammenfügt, dann kann nicht
ausgeschlossen werden, das es "keine" wohlordnung gibt !
weil: wir wissen nicht, wieviel unendlich viele symbolische oo-keit-
en es gibt - das wird wohl auch auf 1000 Jahre so bleiben.
Es sei denn, der liebe Herr hinter unseren Universum oder außerhalb
unseren schwarzen Loches kommt mal vorbei, und verrrät uns das.
Was ich aber sehr startk bezweifeln würde.

> Im allgemeinen kann man annehmen, dass man in einer Arbeit, die
> seit mehr als 100 Jahren jedem Mathematiker bekannt ist, keine
> dermaßen elementare Fehler finden wird.

falsche denkweise !!!
vor 100 Jahren wussten sehr sehr wenige, das es Quarks und Co. gibt.

JVR

unread,
Feb 9, 2024, 2:34:58 AMFeb 9
to
Das erste Zitat stammt aus der Einleitung von Borel's Büchlein; das zweite
aus einer Besprechung von Bagemihl eben dieses Büchleins.

Der Begriff "nombres inaccessibles" kommt in ähnlichem Zusammenhang
bei Kuratowski vor, sowie bei Sierpinski & Tarski.

So wie Borel den Begriff erklärt ist er ganz bestimmt in Hadamard's Sinn
imprädikativ; schlimmer noch - es ist keine Eigenschaft einer Zahl, denn
jede Zahl, für die man bestimmen möchte, ob sie 'inaccessible' ist, ist
schon durch die 'Verfügbarkeit' nicht-inaccessible.

Dies ist kein mathematisches, sondern ein meta-mathematisches Thema.

WM

unread,
Feb 9, 2024, 5:58:33 AMFeb 9
to
Fritz Feldhase schrieb am Donnerstag, 8. Februar 2024 um 20:57:14 UTC+1:
> On Thursday, February 8, 2024 at 8:28:23 PM UTC+1, Ganzhinterseher wrote:
> > JVR schrieb am Donnerstag, 8. Februar 2024 um 18:39:13 UTC+1:
> > > On Thursday, February 8, 2024 at 4:25:36 PM UTC+1, WM wrote:
> > > >
> > > > Der Beweis enthält die Bestimmung: If now m' were the first element
> > > > of M' which differed from the corresponding element m''. Das setzt
> > > > voraus, dass alle Endsegmente linear durchsucht werden können, auch die
> > > > stets unendlich vielen darin verbleibenden Elemente. Und natürlich
> > > > funktioniert das nur für abzählbare Mengen.
> Offenbar haben alle anderen Mathematiker diesen Fehler bisher übersehen

Offenbar. Trotzdem ist es Fakt, dass nur abzählbare Mengen abzählbar sind, so dass es sinnvoll ist, von einem ersten Element zu sprechen.
>
> Wann dürfen wir mit einer entsprechendne Publikation rechnen?
>
> Die mathematische Community wäre sicher überaus interessiert an Ihrer Entdeckung!

Keineswegs, die Matheologen würden alles unternehmen, um den Unsinn, dem sie anhängen nicht aufzudecken. Sie behaupten ja sogar, zu dumm zu sein, um den Unterschied zwischen potentiell und aktual unendlich zu erkennen, obwohl ihr Leitstern und auch Hilbert und andere Nachfolger ihn immer wieder genau und leicht fasslich erklärt haben.

> > > Was Sie oben schreiben ist keine Begründung.
> > Doch, denn was man auf einen ersten Fehltritt prüfen kann, ist Schritt für Schritt prüfbar und damit abzählbar.
> eine solche flapsige Bemerkung ist meiner Meinung nach nicht wirklich ausreichend, um Ihre Endeckung zu begründen.*)

Nur abzählbare Mengen sind abzählbar. Kann man selbst diese Folgerung der Matheologie opfern?

Gruß, WM


WM

unread,
Feb 9, 2024, 6:03:45 AMFeb 9
to
JVR schrieb am Donnerstag, 8. Februar 2024 um 21:08:48 UTC+1:
> On Thursday, February 8, 2024 at 8:28:23 PM UTC+1, Ganzhinterseher wrote:
> > JVR schrieb am Donnerstag, 8. Februar 2024 um 18:39:13 UTC+1:
> > > On Thursday, February 8, 2024 at 4:25:36 PM UTC+1, WM wrote:
> >
> > > > > > Eine genauere Analyse der ZFC-Mengenlehre ist außerdem sinnlos,
> > denn Zermelo "beweist" seinen Wohlordnungssatz nur für abzählbare Mengen.
> > > > Der Beweis enthält die Bestimmung: If now m' were the first element
> > of M' which differed from the corresponding element m''. Das setzt
> > voraus, dass alle Endsegmente linear durchsucht werden können, auch die
> > stets unendlich vielen darin verbleibenden Elemente. Und natürlich
> > funktioniert das nur für abzählbare Mengen.
> > > Wenn man Literatur zitiert, gibt man normalerweise die Quelle an.
> > Das tat ich im Parallelposting. Aber hier ist die Quelle nochmals: [E.
> > Zermelo: "Beweis, daß jede Menge wohlgeordnet werden kann", Math. Ann.
> > 59 (1904) p. 515]
> > > Was Sie oben schreiben ist keine Begründung.
> > Doch, denn was man auf einen ersten Fehltritt prüfen kann, ist Schritt
> > für Schritt prüfbar und damit abzählbar.
> >
> Sie beziehen sich also auf den ersten Paragraphen auf Seite 515.
> Offenbar haben Sie übersehen, dass die beiden Mengen M'_{gamma} und
> M''_{gamma} wohlgeordnet sind, woraus folgt, dass m' bzw m'' als
> erstes Element einer wohldefinierten Untermenge existieren muss.

Es geht um einen Beweis dafür, dass alle Mengen wohlgeordnet werden können.
>
> Im allgemeinen kann man annehmen, dass man in einer Arbeit, die
> seit mehr als 100 Jahren jedem Mathematiker bekannt ist, keine
> dermaßen elementare Fehler finden wird.

Das hat sich schon mehrfach als falsche Annahme erwiesen. Ich erinnere nur an die Indizierung aller Brüche.
Wenn alle positiven Brüche m/n existieren, dann befinden sich alle in der Matrix

1/1, 1/2, 1/3, 1/4, ...
2/1, 2/2, 2/3, 2/4, ...
3/1, 3/2, 3/3, 3/4, ...
4/1, 4/2, 4/3, 4/4, ...
5/1, 5/2, 5/3, 5/4, ...
... .

Wenn alle natürlichen Zahlen k existieren, dann können wir sie verwenden, um damit die Ganzzahlbrüche m/1 in der ersten Spalte zu indizieren. Bezeichnen wir indizierte Brüche mit X und nicht indizierte mit O, so ergibt sich die Matrix

XOOO...
XOOO...
XOOO...
XOOO...
XOOO...
... .

Gruß, WM

WM

unread,
Feb 9, 2024, 6:12:49 AMFeb 9
to
JVR schrieb am Freitag, 9. Februar 2024 um 08:34:58 UTC+1:

> So wie Borel den Begriff erklärt ist er ganz bestimmt in Hadamard's Sinn
> imprädikativ;

Nein. Jede Zahl wird ohne Bezug auf noch nicht definierte definiert. Beispiel: Cantors Konstruktion der natürlichen Zahlen. Cantor weist ausdrücklich darauf hin.

> schlimmer noch - es ist keine Eigenschaft einer Zahl, denn
> jede Zahl, für die man bestimmen möchte, ob sie 'inaccessible' ist, ist
> schon durch die 'Verfügbarkeit' nicht-inaccessible.

So ist es. Deswegen ist es offenbar so schwer, diesen Gedankengang zu verstehen. Alle Zahlen, die man bestimmen möchte, kann man bestimmen. Trotzdem bleiben in jedem Falle fast alle Zahlen unbestimmt.
>
> Dies ist kein mathematisches, sondern ein meta-mathematisches Thema.

Es ist ein sehr wichtiges Thema der Mathematik, wenn auch für die Anwendung völlig belanglos.

Gruß, WM

JVR

unread,
Feb 9, 2024, 6:25:21 AMFeb 9
to
Schon gut, Mücke - diese Themen sind nichts für einen Wirrkopf.

Andreas Leitgeb

unread,
Feb 9, 2024, 11:47:47 AMFeb 9
to
WM <wolfgang.m...@outlook.de> wrote:
> Wenn alle natürlichen Zahlen k existieren, dann können wir sie
> verwenden, um damit die Ganzzahlbrüche m/1 in der ersten Spalte zu
> indizieren.

Wäre es an der "THA" irgendwie "verboten", anstatt nur die erste Spalte
mit "X" zu versorgen, die Xe in der Hand abwechselnd in die erste und
zweite Spalte zu platzieren? Da du solche Skizzen offenbar magst, hier
mal so ein Ablauf:

XOOOO... XXOOO... XXOOO... XXOOO... XXOOO...
OOOOO... OOOOO... XOOOO... XXOOO... XXOOO...
OOOOO... OOOOO... OOOOO... OOOOO... XOOOO...
OOOOO... OOOOO... OOOOO... OOOOO... OOOOO...
... ... ... ... ...

Würde bei diesem Ansatz die WM-atik schon aussteigen, und
behaupten, dass einem dann irgendwo weit unten im schon
ziemlich dunkeln dann vorzeitig die "X"e ausgehen würden?

Nur ein ganz kleines bisserl neugierig...

Ganzhinterseher

unread,
Feb 9, 2024, 2:35:19 PMFeb 9
to a...@logic.at
On 09.02.2024 17:47, Andreas Leitgeb wrote:
> WM <wolfgang.m...@outlook.de> wrote:
>> Wenn alle natürlichen Zahlen k existieren, dann können wir sie
>> verwenden, um damit die Ganzzahlbrüche m/1 in der ersten Spalte zu
>> indizieren.
>
> Wäre es an der "THA" irgendwie "verboten", anstatt nur die erste Spalte
> mit "X" zu versorgen, die Xe in der Hand abwechselnd in die erste und
> zweite Spalte zu platzieren?

Sleight of hand führt leicht zum Betrug. Zunächst muss sichergestellt
werden, dass genau |ℕ| Indizes verwendet werden. Das geschieht durch
Indizierung einer Spalte oder einer Zeile oder einer Diagonale. Und dann
stellt sich heraus, dass die Indizes nicht vermehrbar sind.

Gruß, WM

Fritz Feldhase

unread,
Feb 9, 2024, 3:22:55 PMFeb 9
to
Herr Mückenheim, warum versuchen Sie nicht, den folgeden Entwurf zu publizieren? Ruhm und Ehre wäre Ihnen gewiss! (Schlagzeile [z. B. in der ZEIT]: "Lehrer an einer TH entdeckt Fehler in einem mehr als 100 Jahre alten Beweis!".)

__Ein Fehler in Zermelos Beweis von W. Mückenheim__

Zermelos Beweis*) enthält die Bestimmung: "If now m' were the first element of M' which differed from the corresponding element m''. Das setzt voraus, dass alle Endsegmente linear durchsucht werden können, auch die stets unendlich vielen darin verbleibenden Elemente. Und natürlich funktioniert das nur für abzählbare Mengen. Denn was man auf einen ersten Fehltritt prüfen kann, ist Schritt für Schritt prüfbar und damit abzählbar. Nicht zu vergessen: Nur abzählbare Mengen sind abzählbar.

Fritz Feldhase

unread,
Feb 9, 2024, 4:24:22 PMFeb 9
to
On Friday, February 9, 2024 at 5:47:47 PM UTC+1, Andreas Leitgeb wrote:

> Würde bei diesem Ansatz die WM-atik schon aussteigen, und
> behaupten, dass einem dann irgendwo weit unten im schon
> ziemlich dunkeln dann vorzeitig die "X"e ausgehen würden?

Gute Frage. Man muss sich fragen, ob in der WM-atik die Folge der ungeraden Zahlen, bzw. wie die Folge der geraden Zahlen NICHT unendlich ist.

Weil, WENN diese Folgen unendlich wären, müsste man sich schon fragen, warum sie nicht zur Indizierung der ersten bzw. zweiten Spalte der unendlichen Matrix verwendet werden dürfen/können.

Also mit anderen Worten, warum man die (unendlich vielen) Elemente der ersten Spalte der Matrix X_1,1, X_2,1, X_3,1, ... NICHT mit den (unendlich vielen) Zahlen 1, 3, 5, 7, ... bzw. die (unendlich vielen) Elemente der zweiten Spalte der Matrix X_1,2, X_2,2, X_3,2, ... NICHT mit den (unendlich vielen) Zahlen 2, 4, 6, 8, ... indizieren kann. Wo genau "hakt" es dabei? Für welches Element der beiden Spalten führt das zu einem Problem/scheitert es.

Viell. kann Herr Mückenheim sich dazu äußern. Schließlich ist er ja, der einzige Expert für die WM-atik. Sein Wort hat Gewicht! (->Ex cathedra)

Vermutlich gibt es in der WM-atik die beiden Bijektionen u: {1, 2, 3, 4, ...} -> {1, 3, 5, 7, ...} definiert durch u(n) = 2*n - 1 (für alle n e {1, 2, 3, 4, ...} und g: {1, 2, 3, 4, ...} -> {2, 4, 6, 8, ...} definiert durch g(n) = 2*n (für alle n e {1, 2, 3, 4, ...} nicht.

Ganzhinterseher

unread,
Feb 9, 2024, 5:22:04 PMFeb 9
to Fritz Feldhase
Fritz Feldhase schrieb am Freitag, 9. Februar 2024 um 21:22:55 UTC+1:
> Herr Mückenheim, warum versuchen Sie nicht, den folgeden Entwurf zu
publizieren? Ruhm und Ehre wäre Ihnen gewiss! (Schlagzeile [z. B. in der
ZEIT]: "Lehrer an einer TH entdeckt Fehler in einem mehr als 100 Jahre
alten Beweis!".)

Ach Fritsche, es ist doch kein Geheimnis, dass die herrschenden
Matheologen entweder dumm oder verlogen sind und so etwas so lange, wie
sie die Macht darüber haben, niemals zulassen würden. Das erweist sich
doch schon an viel simpleren Sachen. Wer nicht zugibt, dass unendliche
Endsegmente keinen leeren Schnitt haben können, der ist verlogen, denn
so blöde, es nicht zu erkennen, kann eigentlich niemand sein.

Gruß, WM

Ganzhinterseher

unread,
Feb 9, 2024, 5:32:25 PMFeb 9
to Fritz Feldhase
On 09.02.2024 22:24, Fritz Feldhase wrote:
> On Friday, February 9, 2024 at 5:47:47 PM UTC+1, Andreas Leitgeb wrote:
>
>> Würde bei diesem Ansatz die WM-atik schon aussteigen, und
>> behaupten, dass einem dann irgendwo weit unten im schon
>> ziemlich dunkeln dann vorzeitig die "X"e ausgehen würden?
>
> Gute Frage. Man muss sich fragen, ob in der WM-atik die Folge der ungeraden Zahlen, bzw. wie die Folge der geraden Zahlen NICHT unendlich ist.

Die Folgen sind unendlich, aber viel kürzer als die Folge der Brüche.
Das erkennt man an meinem Beweis. Ohne den Zwischenschritt der
Indizerung einer Zeile oder Spalte oder Diagonale wäre das zwar auch der
Fall, aber die naiven Matheologen erkennen es nicht.
>
> Weil, WENN diese Folgen unendlich wären, müsste man sich schon fragen, warum sie nicht zur Indizierung der ersten bzw. zweiten Spalte der unendlichen Matrix verwendet werden dürfen/können.
>
> Also mit anderen Worten, warum man die (unendlich vielen) Elemente der ersten Spalte der Matrix X_1,1, X_2,1, X_3,1, ... NICHT mit den (unendlich vielen) Zahlen 1, 3, 5, 7, ... bzw. die (unendlich vielen) Elemente der zweiten Spalte der Matrix X_1,2, X_2,2, X_3,2, ... NICHT mit den (unendlich vielen) Zahlen 2, 4, 6, 8, ... indizieren kann. Wo genau "hakt" es dabei? Für welches Element der beiden Spalten führt das zu einem Problem/scheitert es.

Soweit ich verstanden habe, will AL die Indizes in der Hand behalten,
bis alles fertig ist. Dabei kann man das Fehlen von Indizes nicht
unmittelbar erkennen.
>
> Viell. kann Herr Mückenheim sich dazu äußern. Schließlich ist er ja, der einzige Expert für die WM-atik. Sein Wort hat Gewicht! (->Ex cathedra)
>
> Vermutlich gibt es in der WM-atik die beiden Bijektionen u: {1, 2, 3, 4, ...} -> {1, 3, 5, 7, ...} definiert durch u(n) = 2*n - 1 (für alle n e {1, 2, 3, 4, ...} und g: {1, 2, 3, 4, ...} -> {2, 4, 6, 8, ...} definiert durch g(n) = 2*n (für alle n e {1, 2, 3, 4, ...} nicht.

Dass es sie nicht gibt, ist nicht so trivial wie im Falle der Brüche:
All positive fractions

1/1, 1/2, 1/3, 1/4, ...
2/1, 2/2, 2/3, 2/4, ...
3/1, 3/2, 3/3, 3/4, ...
4/1, 4/2, 4/3, 4/4, ...
...

can be indexed by the Cantor function k = (m + n - 1)(m + n - 2)/2 + m
which attaches the index k to the fraction m/n in Cantor's sequence

1/1, 1/2, 2/1, 1/3, 2/2, 3/1, 1/4, 2/3, 3/2, ...

Its terms can be represented by matrices. When we use as indeXes the
integer fractions m/1 and indicate missing indexes by hOles O, then we
get the matrix M(0) as starting position:

XOOO... XXOO... XXOO... XXXO...
XOOO... OOOO... XOOO... XOOO...
XOOO... XOOO... OOOO... OOOO...
XOOO... XOOO... XOOO... OOOO...
... ... ... ...
M(0) M(2) M(3) M(4)

M(1) is the same as M(0) because index 1 remains at 1/1. In M(2) index 2
from 2/1 has been attached to 1/2. In M(3) index 3 from 3/1 has been
attached to 2/1. In M(4) index 4 from 4/1 has been attached to 1/3.
Successively all fractions of the sequence get indexed. In the limit,
denoted by M(∞), we see no fraction without index remaining. Note that
the only difference to Cantor's enumeration is that Cantor does not
render account for the source of the indices.

It should go without saying that by rearranging the X of M(0) never a
complete covering can be realized. Lossless transpositions cannot suffer
losses.

Gruß, WM

Fritz Feldhase

unread,
Feb 9, 2024, 6:23:18 PMFeb 9
to
On Friday, February 9, 2024 at 11:58:33 AM UTC+1, WM wrote:
> Fritz Feldhase schrieb am Donnerstag, 8. Februar 2024 um 20:57:14 UTC+1:
> > On Thursday, February 8, 2024 at 8:28:23 PM UTC+1, Ganzhinterseher wrote:
> > >
> > > Der Beweis enthält die Bestimmung: If now m' were the first element
> > > of M' which differed from the corresponding element m''. Das setzt
> > > voraus, dass alle Endsegmente linear durchsucht werden können, auch die
> > > stets unendlich vielen darin verbleibenden Elemente. Und natürlich
> > > funktioniert das nur für abzählbare Mengen.
> > >
> > Offenbar haben alle anderen Mathematiker diesen Fehler bisher übersehen
> >
> Offenbar. Trotzdem ist es Fakt

Eben darum die Frage an Sie:

Wann dürfen wir mit einer entsprechendne Publikation rechnen?

Die mathematische Community wäre sicher überaus interessiert an Ihrer Entdeckung! (Vielleicht nicht gerade erfreut, aber duchaus interessiert!)

> > "Was Sie oben schreiben ist keine Begründung." [JvR]

Sie sehen, hier tut ein wissenschaftlicher Artikel, wo Sie alles ausführlich erklären und ausführen, Not!

Auf diese Weise können Sie derartige Zweifel an Ihrer Entdeckung ein für allemal ausräumen! (Es wird Zeit, dass Ihren "Kritikern" einmal so richtig das Maul gestopft wird!)

Bitte halten Sie mit einer ausführlichen Begründung nicht länger hinter dem Berg. Die Welt wartet auf Ihren Beitrag!

Ich habe da schon mal einen ersten Entwurf für Sie vorbereitet:

| _Ein Fehler in Zermelos Beweis von W. Mückenheim__
|
| Zermelos Beweis*) enthält die Bestimmung: "If now m' were the first element of M' which differed from the corresponding element m''.
| Das setzt voraus, dass alle Endsegmente linear |durchsucht werden können, auch die stets unendlich vielen darin verbleibenden Elemente.
| Und natürlich funktioniert das nur für abzählbare Mengen. Denn was man auf einen ersten Fehltritt prüfen kann, ist Schritt für Schritt
| prüfbar und damit abzählbar. Nicht zu vergessen: Nur abzählbare Mengen sind abzählbar.

Andreas Leitgeb

unread,
Feb 9, 2024, 6:39:52 PMFeb 9
to
Ganzhinterseher <wolfgang.m...@tha.de> wrote:
> Zunächst muss sichergestellt werden, dass genau |ℕ| Indizes verwendet
> werden.

Wieviele sind das jetzt eigentlich genau?
Ist das zum Beispiel die gleiche Anzahl, wie jene von dezimalen
Darstellungen, also "1", "2", ..., "9", "10", ..., "99", ... ?

Wenn mir die mal nicht reichen, kann ich es dann mit hexadezimalen
Zahlen versuchen? Offenbar(*) gibt es davon ja mehr, weil jede
Dezimalzahl auch als Hexadezimalzahl gelesen werden kann, aber
nicht jede Hexadezimalzahl als Dezimalzahl - Da kann es also dann
auch keine WM-atische Gleichzahligkeit dazwischen geben.

> Und dann stellt sich heraus, dass die Indizes nicht vermehrbar sind.

Wenn mir also die Dezimalzahlen beim nicht-geradlinigen Durchschreiten
der "Matrix" ausgehen, kann ich ja mal schauen, ob ich dann mit
Hexadezimalzahlen (zumindest jenen mit zumindest einer nicht-
dezimalen "Ziffer") weiterzählen kann... Falls die auch nicht
reichen, muss ich halt eine noch größere Basis nehmen.
Welche Basis reicht dann, um z.B. drei "Matrix"-spalten abzudecken?


--
(*): zumindest nach WM-atischem Gleichzahligkeitsprinzip

Fritz Feldhase

unread,
Feb 9, 2024, 6:54:44 PMFeb 9
to
On Friday, February 9, 2024 at 11:32:25 PM UTC+1, Ganzhinterseher wrote:

The elements in the "infinite matrix"

> 1/1, 1/2, 1/3, 1/4, ...
> 2/1, 2/2, 2/3, 2/4, ...
> 3/1, 3/2, 3/3, 3/4, ...
> 4/1, 4/2, 4/3, 4/4, ...
> ...
>
> can be indexed by the Cantor function k = (m + n - 1)(m + n - 2)/2 + m
> which attaches the index k to the fraction m/n [at pos. m,n in the matrix]

Right!

Leading to

> Cantor's sequence
>
> 1/1, 1/2, 2/1, 1/3, 2/2, 3/1, 1/4, 2/3, 3/2, ...

Exactly!

HIER können wir nun aufhören, Mückenheim. DENN DAMIT ist die Abzählbarkeit der Brüche gezeigt, wie Cantor schon erkannt hat:

„Es hat nämlich die Funktion μ + ((μ + ν − 1) (μ + ν − 2))/2, wie leicht zu zeigen, die bemerkenswerte Eigenschaft, dass sie alle positiven [Brüche] und jede[n] nur einmal darstellt, wenn in ihr μ und ν unabhängig voneinander ebenfalls jeden positiven, ganzzahligen Wert erhalten.“ (G. Cantor).

Der Rest Deines Geschwalles ist nicht weiter von Interesse.

Ganzhinterseher

unread,
Feb 10, 2024, 6:34:29 AMFeb 10
to a...@logic.at
On 10.02.2024 00:39, Andreas Leitgeb wrote:
> Ganzhinterseher <wolfgang.m...@tha.de> wrote:
>> Zunächst muss sichergestellt werden, dass genau |ℕ| Indizes verwendet
>> werden.
>
> Wieviele sind das jetzt eigentlich genau?

|ℕ| ist größer als alle definierbaren Zahlen n und außerdem alle dunklen
Zahlen. |ℕ| ist kleiner als |ℕ| +1. Mehr ist darüber nicht bekannt.

> Ist das zum Beispiel die gleiche Anzahl, wie jene von dezimalen
> Darstellungen, also "1", "2", ..., "9", "10", ..., "99", ... ?

Du sprichst ein interessantes Thema an. Bisher habe ich
gedankenloserweise die natürlichen Zahlen als basisunabhängig betrachtet
und eine Basisabhängigkeit nur für reelle Zahlen angenommen. Die
Darstellungsweise entscheidet aber auch hier über die Anzahl. Die
Unärdarstellung I, II, III, ... oder
1, 11, 111, ...
enthält alle natürlichen Zahlen, aber weniger Zahlen als die
Binärdarstellung
1, 10, 11, 100, 101, 110, 111, ...
oder die Dezimaldarstellung
1, 2, 3, ..., 11, ..., 111, ...
oder Darstellungen in noch größeren Basen.

Für das Problem der Indizierung aller Brüche spielt das aber keine
Rolle, wenn man die Brüche in derselben Basis darstellt

> Wenn mir die mal nicht reichen, kann ich es dann mit hexadezimalen
> Zahlen versuchen? Offenbar(*) gibt es davon ja mehr, weil jede
> Dezimalzahl auch als Hexadezimalzahl gelesen werden kann, aber
> nicht jede Hexadezimalzahl als Dezimalzahl - Da kann es also dann
> auch keine WM-atische Gleichzahligkeit dazwischen geben.

Richtig.

>> Und dann stellt sich heraus, dass die Indizes nicht vermehrbar sind.
>
> Wenn mir also die Dezimalzahlen beim nicht-geradlinigen Durchschreiten
> der "Matrix" ausgehen, kann ich ja mal schauen, ob ich dann mit
> Hexadezimalzahlen (zumindest jenen mit zumindest einer nicht-
> dezimalen "Ziffer") weiterzählen kann... Falls die auch nicht
> reichen, muss ich halt eine noch größere Basis nehmen.
> Welche Basis reicht dann, um z.B. drei "Matrix"-spalten abzudecken?

Es reichen in derselben Basis, wie man sie für die Brüche anwendet, drei
Mengen: ℕ, ℕ' und ℕ''.

Über die Anzahlverhältnisse zwischen verschiedenen Basen muss man noch
nachdenken.

Gruß, WM

Ganzhinterseher

unread,
Feb 10, 2024, 7:14:22 AMFeb 10
to a...@logic.at
Fritz Feldhase schrieb am Samstag, 10. Februar 2024 um 00:54:44 UTC+1:
> On Friday, February 9, 2024 at 11:32:25 PM UTC+1, Ganzhinterseher wrote:
>
> The elements in the "infinite matrix"
> > 1/1, 1/2, 1/3, 1/4, ...
> > 2/1, 2/2, 2/3, 2/4, ...
> > 3/1, 3/2, 3/3, 3/4, ...
> > 4/1, 4/2, 4/3, 4/4, ...
> > ...
> >
> > can be indexed by the Cantor function k = (m + n - 1)(m + n - 2)/2 + m
> > which attaches the index k to the fraction m/n [at pos. m,n in the
matrix]
>
> Right!
>
> Leading to
> > Cantor's sequence
> >
> > 1/1, 1/2, 2/1, 1/3, 2/2, 3/1, 1/4, 2/3, 3/2, ...
> Exactly!
>
> HIER können wir nun aufhören, Mückenheim. DENN DAMIT ist die
Abzählbarkeit der Brüche gezeigt, wie Cantor schon erkannt hat:
>
> „Es hat nämlich die Funktion μ + ((μ + ν − 1) (μ + ν − 2))/2, wie
leicht zu zeigen, die bemerkenswerte Eigenschaft, dass sie alle
positiven [Brüche] und jede[n] nur einmal darstellt, wenn in ihr μ und ν
unabhängig voneinander ebenfalls jeden positiven, ganzzahligen Wert
erhalten.“ (G. Cantor).

Und das ist falsch.
>
> Der Rest ist nicht weiter von Interesse.

Du weigerst Dich darüber nachzudenken, dass in jedem endlichen Schritt n
die Matrix M(n) kein einziges O verloren hat. Im Grenzfalle wird aber
nichts indiziert.

Gruß, WM

Andreas Leitgeb

unread,
Feb 10, 2024, 8:21:37 AMFeb 10
to
Ganzhinterseher <wolfgang.m...@tha.de> wrote:
>> Ist das zum Beispiel die gleiche Anzahl, wie jene von dezimalen
>> Darstellungen, also "1", "2", ..., "9", "10", ..., "99", ... ?
>
> Du sprichst ein interessantes Thema an.

Für einen Neuling der *Mathematik* mag es das wohl sein.

Jedenfalls hast du (auch wenn du dir dessen formal nicht bewusst warst)
die Gleichzahligkeit durch willkürliche Einschränkung "erlaubter"
Bijektionen zu erhalten versucht. Somit war die Menge der natürlichen
Zahlen {1,2,3,...} "gleichzahlig" mit der Menge der Zahlen/jeweils
mit "*2" oder auch "0" angehängt (aber "ohne Ausrechnen"), somit also
etwa mit {1"0", 2"0", 3"0",...}, aber nicht mit {10, 20, 30,...}
Danke noch für die damit verbundene Belustigung des Publikums.

Die verschiedenen Zahlenbasen sind nun der Todesstoß für dieses
von vornherein fehlerhafte Konzept der "Gleichzahligkeit",
da ℕ offenbar nun "gleichzahlig" zu untereinander nicht-"gleich-
zahligen" Mengen ist, nämlich als da zum Beispiel wären: die Menge der
binär-, dezimal- und hexadezimal-darstellungen natürlicher Zahlen.

> Über die Anzahlverhältnisse zwischen verschiedenen Basen muss man noch
> nachdenken.

Ja bitte bleib an dem Gedanken dran, dann landest du vielleicht
doch noch bei der Erkenntnis, dass "Gleichzahligkeit" für unendliche
Mengen widersprüchlich & sinnlos ist, und deswegen (seit Cantor)
durch das neue Konzept der "Gleichmächtigkeit" auf Basis von
Bijektionen ersetzt wurde.

Fritz Feldhase

unread,
Feb 10, 2024, 8:26:45 AMFeb 10
to
On Saturday, February 10, 2024 at 1:14:22 PM UTC+1, Ganzhinterseher wrote:
> Fritz Feldhase schrieb am Samstag, 10. Februar 2024 um 00:54:44 UTC+1:
> >
> > "Es hat nämlich die Funktion μ + ((μ + ν − 1) (μ + ν − 2))/2, wie
> > leicht zu zeigen, die bemerkenswerte Eigenschaft, dass sie alle
> > positiven [Brüche] und jede[n] nur einmal darstellt, wenn in ihr μ und ν
> > unabhängig voneinander ebenfalls jeden positiven, ganzzahligen Wert
> > erhalten." (G. Cantor).
> >
> Das ist falsch.

Ach? Können Sie das bitte etwas genauer ausführen, Müchenheim?

Hat Ihrer Meinung nach die "Funktion μ + ((μ + ν − 1) (μ + ν − 2))/2" NICHT "die bemerkenswerte Eigenschaft, dass sie alle positiven [Brüche] und jede[n] nur einmal darstellt, wenn in ihr μ und ν unabhängig voneinander ebenfalls jeden positiven, ganzzahligen Wert erhalten"?

Please elaborate!

Ganzhinterseher

unread,
Feb 10, 2024, 9:32:41 AMFeb 10
to Fritz Feldhase
On 10.02.2024 14:26, Fritz Feldhase wrote:
> On Saturday, February 10, 2024 at 1:14:22 PM UTC+1, Ganzhinterseher wrote:
>> Fritz Feldhase schrieb am Samstag, 10. Februar 2024 um 00:54:44 UTC+1:
>>>
>>> "Es hat nämlich die Funktion μ + ((μ + ν − 1) (μ + ν − 2))/2, wie
>>> leicht zu zeigen, die bemerkenswerte Eigenschaft, dass sie alle
>>> positiven [Brüche] und jede[n] nur einmal darstellt, wenn in ihr μ und ν
>>> unabhängig voneinander ebenfalls jeden positiven, ganzzahligen Wert
>>> erhalten." (G. Cantor).
>>>
>> Das ist falsch.
>
> Ach? Können Sie das bitte etwas genauer ausführen

Das tat ich doch schon oft.

Wenn Cantor recht hätte, dann müsste es möglich sein, durch beliebiges
Verschieben der X in der Matrix
XOOO...
XOOO...
XOOO...
XOOO...
...
diese mit X vollständig zu überdecken, und zwar im Verlaufe der endlich
indizierten Schritte, nicht etwa "im Grenzfall".

Du wirst es natürlich löschen, weil Du kein einziges Gegenargument hast,
Deinen Glauben aber nicht aufgeben willst. Da hilft nur Löschung der
Blasphemie.

Gruß, WM

Fritz Feldhase

unread,
Feb 10, 2024, 9:45:50 AMFeb 10
to
On Saturday, February 10, 2024 at 3:32:41 PM UTC+1, Ganzhinterseher wrote:
> On 10.02.2024 14:26, Fritz Feldhase wrote:
> > On Saturday, February 10, 2024 at 1:14:22 PM UTC+1, Ganzhinterseher wrote:
> >> Fritz Feldhase schrieb am Samstag, ⁷ UTC+1:
> >>>
> >>> "Es hat nämlich die Funktion μ + ((μ + ν − 1) (μ + ν − 2))/2, wie
> >>> leicht zu zeigen, die bemerkenswerte Eigenschaft, dass sie alle
> >>> positiven [Brüche] und jede[n] nur einmal darstellt, wenn in ihr μ und ν
> >>> unabhängig voneinander ebenfalls jeden positiven, ganzzahligen Wert
> >>> erhalten." (G. Cantor).
> >>>
> > > Das ist falsch.
> > >
> > Ach? Können Sie das bitte etwas genauer ausführen, Mückenheim?
> >
> Das tat ich doch schon oft.

Nein, das haben Sie noch nicht getan. Sie sollen hier nicht über irgend einen von ihnen selbst produzierten Scheißdreck schwadronieren, sondern erklären, w a s genau an Cantors Aussage/Behauptung falsch ist. Ich habe sie dazu gefragt:

Hat also Ihrer Meinung nach die "Funktion μ + ((μ + ν − 1) (μ + ν − 2))/2" NICHT "die bemerkenswerte Eigenschaft, dass sie alle positiven [Brüche] und jede[n] nur einmal darstellt, wenn in ihr μ und ν unabhängig voneinander ebenfalls jeden positiven, ganzzahligen Wert erhalten"?

Please elaborate!

Ganzhinterseher

unread,
Feb 10, 2024, 9:47:39 AMFeb 10
to a...@logic.at
On 10.02.2024 14:21, Andreas Leitgeb wrote:
> Ganzhinterseher <wolfgang.m...@tha.de> wrote:

> Jedenfalls hast du (auch wenn du dir dessen formal nicht bewusst warst)
> die Gleichzahligkeit durch willkürliche Einschränkung "erlaubter"
> Bijektionen zu erhalten versucht.


Da ist nichts wollkürlich an der Tatsache, dass weniger Ganzzahlen als
Brüche existieren.
Wenn Cantor recht hätte, dann müsste es möglich sein, durch beliebiges
Verschieben der X in der Matrix
XOOO...
XOOO...
XOOO...
XOOO...
...
diese mit X vollständig zu überdecken, und zwar im Verlaufe der endlich
indizierten Schritte, nicht etwa "im Grenzfall".


> Somit war die Menge der natürlichen
> Zahlen {1,2,3,...} "gleichzahlig" mit der Menge der Zahlen/jeweils
> mit "*2" oder auch "0" angehängt (aber "ohne Ausrechnen"), somit also
> etwa mit {1"0", 2"0", 3"0",...}, aber nicht mit {10, 20, 30,...}

Dagegen ist doch nichts zu sagen. Bei gleicher Basis gibt es nun einmal
eine feste Anzahl natürlicher Zahlen, zu der keine Zahl mehr hinzugefügt
werden kann.

> Die verschiedenen Zahlenbasen sind nun der Todesstoß für dieses
> von vornherein fehlerhafte Konzept der "Gleichzahligkeit",
> da ℕ offenbar nun "gleichzahlig" zu untereinander nicht-"gleich-
> zahligen" Mengen ist, nämlich als da zum Beispiel wären: die Menge der
> binär-, dezimal- und hexadezimal-darstellungen natürlicher Zahlen.

Es geht bei der Messung von Zahldarstellungen um Zahlendarstellungen. Es
gibt ja auch viel mehr Brüche als rationale Zahlen.
>
>> Über die Anzahlverhältnisse zwischen verschiedenen Basen muss man noch
>> nachdenken.
>
> Ja bitte bleib an dem Gedanken dran, dann landest du vielleicht
> doch noch bei der Erkenntnis, dass "Gleichzahligkeit" für unendliche
> Mengen widersprüchlich & sinnlos ist, und deswegen (seit Cantor)
> durch das neue Konzept der "Gleichmächtigkeit" auf Basis von
> Bijektionen ersetzt wurde.

Die hat sich als fehlerhaft erwiesen, denn zum Beispiel muss die
inklusionsmonotone Folge unendlicher Endsegmente einen leeren Schnitt
haben, was nicht möglich ist.

Hint: Es geht nicht um Quatorenvertauschung, sondern nur um die Menge
aller unendlichen Endsegmente, die man aufgrund eben dieser Eigenschaft
identifizieren und deren Schnitt man bilden kann.

Gruß, WM

Ganzhinterseher

unread,
Feb 10, 2024, 9:58:22 AMFeb 10
to Fritz Feldhase
Fritz Feldhase schrieb am Samstag, 10. Februar 2024 um 15:45:50 UTC+1:
> On Saturday, February 10, 2024 at 3:32:41 PM UTC+1, Ganzhinterseher
wrote:

Wenn Cantor recht hätte, dann müsste es möglich sein, durch beliebiges
Verschieben der X in der Matrix
XOOO...
XOOO...
XOOO...
XOOO...
...
diese mit X vollständig zu überdecken, und zwar im Verlaufe der endlich
indizierten Schritte, nicht etwa "im Grenzfall"

> Sie sollen hier nicht über irgend einen von ihnen selbst produzierten

Was verstehst Du daran nicht?
Ist es die Bijektion n <--> n/1?

> Hat also Ihrer Meinung nach die "Funktion μ + ((μ + ν − 1) (μ + ν −
2))/2" NICHT "die bemerkenswerte Eigenschaft, dass sie alle positiven
[Brüche] und jede[n] nur einmal darstellt, wenn in ihr μ und ν
unabhängig voneinander ebenfalls jeden positiven, ganzzahligen Wert
erhalten"?

Die Funktion stellt natürlich nicht alle positiven Brüche dar, denn
wollte man es beweisen, so würde man feststellen, dass auf jeden
unendlich viele folgen, für die man nicht beweisen kann, dass diese
dargestellt werden.

Man muss es also *glauben*, dass die Funktion alle Brüche umfasst. Doch
dieser Glaube wird durch meinen Beweis *widerlegt*.

Gruß, WM

Fritz Feldhase

unread,
Feb 10, 2024, 10:47:20 AMFeb 10
to
On Saturday, February 10, 2024 at 12:54:44 AM UTC+1, Fritz Feldhase wrote:
> On Friday, February 9, 2024 at 11:32:25 PM UTC+1, Ganzhinterseher wrote:
>
> The elements in the "infinite matrix"
> >
> > 1/1, 1/2, 1/3, 1/4, ...
> > 2/1, 2/2, 2/3, 2/4, ...
> > 3/1, 3/2, 3/3, 3/4, ...
> > 4/1, 4/2, 4/3, 4/4, ...
> > ...
> >
> > can be indexed by the Cantor function k = (m + n - 1)(m + n - 2)/2 + m
> > which attaches the index k to the fraction m/n [at pos. m,n in the matrix]
>
> Right!
>
> Leading to
> >
> > Cantor's sequence
> >
> > 1/1, 1/2, 2/1, 1/3, 2/2, 3/1, 1/4, 2/3, 3/2, ...
> >
> Exactly!
>
> HIER können wir nun aufhören, Mückenheim. DENN DAMIT ist die Abzählbarkeit der Brüche gezeigt, wie Cantor schon erkannt hat:

Korrektur:

> „Es hat nämlich die Funktion μ + ((μ + ν − 1) (μ + ν − 2))/2, wie leicht zu zeigen, die bemerkenswerte Eigenschaft, dass sie alle positiven ganzen Zahlen und jede nur einmal darstellt, wenn in ihr μ und ν unabhängig voneinander ebenfalls jeden positiven, ganzzahligen Wert erhalten.“ (G. Cantor).

Fritz Feldhase

unread,
Feb 10, 2024, 10:48:45 AMFeb 10
to
On Saturday, February 10, 2024 at 3:32:41 PM UTC+1, Ganzhinterseher wrote:
> On 10.02.2024 14:26, Fritz Feldhase wrote:
> > On Saturday, February 10, 2024 at 1:14:22 PM UTC+1, Ganzhinterseher wrote:
> >> Fritz Feldhase schrieb am Samstag, 10. Februar 2024 um 00:54:44 UTC+1:

Korrigiert:

> >>> "Es hat nämlich die Funktion μ + ((μ + ν − 1) (μ + ν − 2))/2, wie
> >>> leicht zu zeigen, die bemerkenswerte Eigenschaft, dass sie alle
> >>> positiven ganzen Zahlen und jede nur einmal darstellt, wenn in ihr μ und ν
> >>> unabhängig voneinander ebenfalls jeden positiven, ganzzahligen Wert
> >>> erhalten." (G. Cantor).
> >>>
> >> Das ist falsch.
> >
> > Ach? Können Sie das bitte etwas genauer ausführen?
> >
> Das tat ich doch schon oft.

Nope.

Fritz Feldhase

unread,
Feb 10, 2024, 10:51:34 AMFeb 10
to
On Saturday, February 10, 2024 at 3:58:22 PM UTC+1, Ganzhinterseher wrote:
> Fritz Feldhase schrieb am Samstag, 10. Februar 2024 um 15:45:50 UTC+1:

Korrektur:

> > Hat also Ihrer Meinung nach die "Funktion μ + ((μ + ν − 1) (μ + ν −
> > 2))/2" NICHT "die bemerkenswerte Eigenschaft, dass sie alle positiven
> > ganzen Zahlen und jede nur einmal darstellt, wenn in ihr μ und ν
> > unabhängig voneinander ebenfalls jeden positiven, ganzzahligen Wert
> > erhalten"?

Please elaborate.

Andreas Leitgeb

unread,
Feb 10, 2024, 4:17:28 PMFeb 10
to
Ganzhinterseher <wolfgang.m...@tha.de> wrote:
> Was verstehst Du daran nicht?
> Ist es die Bijektion n <--> n/1?

Ja.
Genaugenommen ist es das: was wird denn nun den "überschüssigen"
Hexadezimalzahlen zugeordnet?

Andreas Leitgeb

unread,
Feb 10, 2024, 4:24:48 PMFeb 10
to
Ganzhinterseher <wolfgang.m...@tha.de> wrote:
> Es geht bei der Messung von Zahldarstellungen um Zahlendarstellungen.
> Es gibt ja auch viel mehr Brüche als rationale Zahlen.

Aber wie ist das nun, wenn man den dezimal-darstellungen der natürlichen
Zahlen noch jene hexadezimalen Zahlen, die nicht auch dezimal sind, zur
Seite stellt? Reicht das dann vielleicht schon für die Brüche?

>> Ja bitte bleib an dem Gedanken dran, dann landest du vielleicht
>> doch noch bei der Erkenntnis, dass "Gleichzahligkeit" für unendliche
>> Mengen widersprüchlich & sinnlos ist ...

Das ist noch offen.
Du hast dich in deinen Gedanken zu weit ablenken lassen, und bist dabei
wieder durchs logische Nirvana gezogen.

Also bitte: bei der Sache bleiben.

Fritz Feldhase

unread,
Feb 10, 2024, 7:02:10 PMFeb 10
to
On Saturday, February 10, 2024 at 3:47:39 PM UTC+1, Ganzhinterseher wrote:
>
> zum Beispiel muss die inklusionsmonotone Folge unendlicher Endsegmente einen leeren Schnitt
> haben,

Hat sie ja auch.

> was nicht möglich ist.

Ach?

W e l c h e natürliche Zahl ist denn in diesem Schnitt enthalten, wenn es für j e d e natürliche Zahl einen (unendlichen) Endabschnitt gibt, in dem sie n i c h t enthalten ist?

Please elaborate.

Ralf Bader

unread,
Feb 11, 2024, 2:00:20 AMFeb 11
to
On 02/10/2024 10:24 PM, Andreas Leitgeb wrote:
> Ganzhinterseher <wolfgang.m...@tha.de> wrote:
>> Es geht bei der Messung von Zahldarstellungen um Zahlendarstellungen.
>> Es gibt ja auch viel mehr Brüche als rationale Zahlen.
>
> Aber wie ist das nun, wenn man den dezimal-darstellungen der natürlichen
> Zahlen noch jene hexadezimalen Zahlen, die nicht auch dezimal sind, zur
> Seite stellt? Reicht das dann vielleicht schon für die Brüche?

Man kann die mit 1 beginnenden Wörter endlicher Länge über dem Alphabet
{0,1} auffassen einerseits als Binärdarstellungen natürlicher Zahlen,
andererseits aber auch als Darstellungen rationaler Zahlen bzw. Brüchen,
wobei die letzte 1 in so einem Wort die Rolle des Bruchstrichs
übernimmt, der Zähler ist in Binärdarstellung durch das Teilwort links
dieser 1 gegeben, der Nenner durch die Anzahl der auf diese 1 folgenden
Nullen. Randfälle, wenn etwa die letzte 1 das Wort beendet, können
relativ zwanglos separat verarztet und einsortiert werden. Ich habe das
irgendwann letztes Jahr hier genauer ausgeführt, ohne daß es auf
Beachtung gestoßen wäre.

JVR

unread,
Feb 11, 2024, 3:21:26 AMFeb 11
to
Wenn man {p/q} schreibt, wobei p and q natürliche Zahlen sind,
stellt man damit die Brüche dar.
Wenn man den Bruchstrich / weg lässt, kann man {pq} als eine Untermenge
der natürlichen Zahlen deuten.

In Mückenhausen sagt man daher: Es gibt mehr natürliche Zahlen als Brüche.

Damit muss sogar der Prefosser einsehen, dass sein und Sponsels "genau so viele",
"mehr" und "weniger" widersprüchlich sind. Und vielleicht sogar, wieso
Cantor den Begriff umdefiniert hat.

Denn bald kommt der Osterhase und etwas später der Weihnachtsmann, auch
Sankt Nikolaus genannt, und droht Prefosser McBughouse mit der Rute, auf dass
er lerne, seine Sturheit zu überwinden.

Denn so doof wie McBughouse zu sein ist verboten.

Carlo XYZ

unread,
Feb 11, 2024, 3:51:46 AMFeb 11
to
Ralf Bader schrieb am 11.02.24 um 08:00:
> On 02/10/2024 10:24 PM, Andreas Leitgeb wrote:
>> Ganzhinterseher <wolfgang.m...@tha.de> wrote:
>>> Es geht bei der Messung von Zahldarstellungen um Zahlendarstellungen.
>>> Es gibt ja auch viel mehr Brüche als rationale Zahlen.
>>
>> Aber wie ist das nun, wenn man den dezimal-darstellungen der natürlichen
>> Zahlen noch jene hexadezimalen Zahlen, die nicht auch dezimal sind, zur
>> Seite stellt?  Reicht das dann vielleicht schon für die Brüche?
>
> Man kann die mit 1 beginnenden Wörter endlicher Länge über dem Alphabet
> {0,1} auffassen einerseits als Binärdarstellungen natürlicher Zahlen,
> andererseits aber auch als Darstellungen rationaler Zahlen bzw. Brüchen,
> wobei die letzte 1 in so einem Wort die Rolle des Bruchstrichs
> übernimmt, der Zähler ist in Binärdarstellung durch das Teilwort links
> dieser 1 gegeben, der Nenner durch die Anzahl der auf diese 1 folgenden
> Nullen. Randfälle, wenn etwa die letzte 1 das Wort beendet, können
> relativ zwanglos separat verarztet und einsortiert werden. Ich habe das
> irgendwann letztes Jahr hier genauer ausgeführt, ohne daß es auf
> Beachtung gestoßen wäre.

Wahrscheinlich, weil das ein alter Hut ist, unter dem
sich ein langer weißer Zopf verbirgt, der, aufgerollt,
bis in den Keller reichen würde (Stichwort Codierung).

Außerdem waren Andreas' Fragen rhetorisch gemeint und
dienten der (mit Sicherheit allerdings vergeblichen)
Erziehung unseres berüchtigten Professors.

WM

unread,
Feb 11, 2024, 5:41:05 AMFeb 11
to
Wenn alles in Hexas dargestellt wird, dann gibt es keine überschüssigen.

Gruß, WM

WM

unread,
Feb 11, 2024, 5:50:07 AMFeb 11
to
In allen unendlichen Endsegmenten sind unendlich viele Zahlen enthalten. Deren Schnitt ist unendlich. Was sollte er sonst sein? Bis zu jedem endlichen k gilt das jedenfalls
∀k ∈ ℕ: ∩{E(1), E(2), ..., E(k)} = E(k) /\ |E(k)| = ℵ₀
also für alle k Deines Beweises
∀k ∈ ℕ: E(k+1) = E(k) \ {k}.
Und danach folgt nichts Definierbares mehr.

Gruß, WM

Ganzhinterseher

unread,
Feb 11, 2024, 6:10:33 AMFeb 11
to Ralf Bader

Ralf Bader schrieb am Sonntag, 11. Februar 2024 um 08:00:20 UTC+1:

> Man kann die mit 1 beginnenden Wörter endlicher Länge über dem Alphabet
> {0,1} auffassen einerseits als Binärdarstellungen natürlicher Zahlen,
> andererseits aber auch als Darstellungen rationaler Zahlen bzw. Brüchen,
> wobei die letzte 1 in so einem Wort die Rolle des Bruchstrichs
> übernimmt, der Zähler ist in Binärdarstellung durch das Teilwort links
> dieser 1 gegeben, der Nenner durch die Anzahl der auf diese 1 folgenden
> Nullen.

Das ändert nichts daran, dass die Cantorsche Methode versagt:
XOOO...
XOOO...
XOOO...
XOOO...
...
Hier kann man die X beliebig, nach Cantor oder anders, mit O
vertauschen, ohne dass ein O weniger resultiert. Damit ist Cantor
widerlegt. Dass andere Argumente anderes ergeben, ändert daran überhaupt
nichts. Es spricht allenfalls dafür, dass dunkle Zahlen existieren und
sich der näheren Analyse entziehen. Aber wie Fritsche und andere
Matheologen hier, das Argument einfach nicht zur Kenntnis zu nehmen,
zeugt von Fanatismus, Dogmatismus und vor allem unwissenschaftlichem
Benehmen.

Gruß, WM

Ralf Bader

unread,
Feb 11, 2024, 10:53:31 AMFeb 11
to
On 02/11/2024 09:51 AM, Carlo XYZ wrote:
> Ralf Bader schrieb am 11.02.24 um 08:00:
>> On 02/10/2024 10:24 PM, Andreas Leitgeb wrote:
>>> Ganzhinterseher <wolfgang.m...@tha.de> wrote:
>>>> Es geht bei der Messung von Zahldarstellungen um Zahlendarstellungen.
>>>> Es gibt ja auch viel mehr Brüche als rationale Zahlen.
>>>
>>> Aber wie ist das nun, wenn man den dezimal-darstellungen der natürlichen
>>> Zahlen noch jene hexadezimalen Zahlen, die nicht auch dezimal sind, zur
>>> Seite stellt? Reicht das dann vielleicht schon für die Brüche?
>>
>> Man kann die mit 1 beginnenden Wörter endlicher Länge über dem
>> Alphabet {0,1} auffassen einerseits als Binärdarstellungen natürlicher
>> Zahlen, andererseits aber auch als Darstellungen rationaler Zahlen
>> bzw. Brüchen, wobei die letzte 1 in so einem Wort die Rolle des
>> Bruchstrichs übernimmt, der Zähler ist in Binärdarstellung durch das
>> Teilwort links dieser 1 gegeben, der Nenner durch die Anzahl der auf
>> diese 1 folgenden Nullen. Randfälle, wenn etwa die letzte 1 das Wort
>> beendet, können relativ zwanglos separat verarztet und einsortiert
>> werden. Ich habe das irgendwann letztes Jahr hier genauer ausgeführt,
>> ohne daß es auf Beachtung gestoßen wäre.
>
> Wahrscheinlich, weil das ein alter Hut ist, unter dem
> sich ein langer weißer Zopf verbirgt, der, aufgerollt,
> bis in den Keller reichen würde (Stichwort Codierung).

Wie es im Rahmen des hier stattfindenden saudummen Geschwafels, das sich
wie ein Mühlrad in den immergleichen Windungen dreht, so etwas wie einen
alten Hut geben soll, ist allerdings ein echtes Rätsel.

Ralf Bader

unread,
Feb 11, 2024, 10:59:24 AMFeb 11
to
Das ist nicht "Cantors Methode", sondern eine Ihrer zahlreichen, oder
eher zahllosen, Wahnideen. Natürlich verschwindet keine 0 durch
Vertauschen mit den X, sondern sie ändert nur ihren Platz. Mückenheim,
Sie sind offenkundig vollkommen unfähig, den Sinn der von Ihnen manisch
beschwafelten Dinge auch nur ansatzweise zu erfassen.


Message has been deleted

Fritz Feldhase

unread,
Feb 11, 2024, 11:53:59 AMFeb 11
to
On Sunday, February 11, 2024 at 11:50:07 AM UTC+1, WM wrote:
> Fritz Feldhase schrieb am Sonntag, 11. Februar 2024 um 01:02:10 UTC+1:
> >
> > W e l c h e natürliche Zahl ist denn im Schnitt über alle (unendlichen) Endsegmente enthalten,
> > wenn es für j e d e natürliche Zahl einen (unendlichen) Endabschnitt gibt, in dem sie n i c h t
> > enthalten ist?
> >
> In allen unendlichen Endsegmenten sind unendlich viele Zahlen enthalten.

Äh ja, danke, aber DANACH hatte ich nicht gefragt. Meine Frage war:

W e l c h e natürliche Zahl ist denn im Schnitt über alle (unendlichen) Endsegmente enthalten, wenn es für j e d e natürliche Zahl einen (unendlichen) Endabschnitt gibt, in dem sie n i c h t enthalten ist?

Please elaborate.

> Deren Schnitt ist unendlich.

Nö.

> Was sollte er sonst sein?

Leer.

Da Sie das offenbar anders sehen, hier nocheinmal meine Frage dazu:

W e l c h e natürliche Zahl ist denn im Schnitt über alle (unendlichen) Endsegmente enthalten, wenn es für j e d e natürliche Zahl einen (unendlichen) Endabschnitt gibt, in dem sie n i c h t enthalten ist?

Oder anders: W i e k a n n eine natürliche Zahl im Schnitt über alle (unendlichen) Endsegmente enthalten sein, wenn es für j e d e natürliche Zahl einen (unendlichen) Endabschnitt gibt, in dem sie n i c h t enthalten ist?

Please elaborate.

Andreas Leitgeb

unread,
Feb 11, 2024, 2:18:31 PMFeb 11
to
Jaja, aber was, wenn nicht?

Hast du schon darüber nachgedacht, wie das möglich ist, dass die Menge
der natürlichen Zahlen mit jeder Basis-darstellung "gleichzahlig" sein
soll, wenn die Mengen der Basis-darstellungen untereinander für
verschiedene Basen nicht "gleichzahlig" sind?

Und wie geht es dir beim Gedanken daran, dass durch geschickte Wahl der
Darstellung die Menge der Bruch-darstellungen eine echte Teilmenge der
Dezimalzahlen ist?

Jens Kallup

unread,
Feb 11, 2024, 2:35:39 PMFeb 11
to
Am 2024-02-11 um 16:59 schrieb Ralf Bader:
> Das ist nicht "Cantors Methode", sondern eine Ihrer zahlreichen, oder
> eher zahllosen, Wahnideen. Natürlich verschwindet keine 0 durch
> Vertauschen mit den X, sondern sie ändert nur ihren Platz. Mückenheim,
> Sie sind offenkundig vollkommen unfähig, den Sinn der von Ihnen manisch
> beschwafelten Dinge auch nur ansatzweise zu erfassen.

da gibt es doch so ein "Memory" physikallisches Denkspiel, das aus einen
quadratischen Rahmen besteht, und im inneren sind verschiebare, kleinere
quadratische Rähmchen, wobei nicht ALLE verwendet werden und "ein" (1)
Rähmchen fehlt, damit man die Rähmchen verschieben kann.

Meist haben diese Rähmchen eine Nummer aufgedruckt, die in einer bestim-
mten Reihenfolge oder Lage angeordnet werden sollen.

Möglich wäre es, das der gute alte Herr WM dies zum Anlaß nimmt, und das
diagonal Argument von Cantor darauf zu projektzieren, um herauszufinden,
welche Indices an welcher Position auftretten...

Was natürlich von mir im kalten Kaffeesatz gepobbelt ist, und somit ohne
Gewehr und ohne Anspruch auf Garantien ist.

Jens

--
Diese E-Mail wurde von Avast-Antivirussoftware auf Viren geprüft.
www.avast.com

Carlo XYZ

unread,
Feb 11, 2024, 2:37:16 PMFeb 11
to
Ralf Bader schrieb am 11.02.24 um 16:53:
Ich spreche weder vom Urheber dieses Geschwafels (dem ich kaum
zutraue, alten von neuen Hüten unterscheiden zu können) noch von
dessen Perpetuierern. Sondern von einer effektiven, umkehrbaren
Umrechnungsmöglichkeit zwischen Zahlen (inklusive rationalen) und
endlichen Wörtern. Das ist mindestens so alt wie Gödel, eher noch
älter: eine Zahl als Abstraktion von Anzahl und eine Zahl als ein
endliches Wort über einem Ziffernsystem (selbst wenn es nur eine
einzige Ziffer gibt, vulgo Bierdeckelsystem). Das kannten schon
die alten <werauchimmer>. Manche sagen: älter als 40000 Jahre[1].

Jeder Komplexitätstheoretiker erfindet seine eigene Methode der
Umrechnung oder schreibt sie von anderen ab. Das ist prinzipiell
für alle nicht gerade allzu pathologisch definierten abzählbaren
Mengen von Zahlen (von endlichen Wörtern) möglich, insbesondere
für Q, und deine obige Erfindung ist eine unter vielen effektiv
äquivalenten Codierungen. (Effektiv äquivalent = algorithmisch
ineinander umrechenbar).

[1] <https://en.wikipedia.org/wiki/History_of_ancient_numeral_systems>

Carlo XYZ

unread,
Feb 11, 2024, 3:00:02 PMFeb 11
to
Carlo XYZ schrieb am 11.02.24 um 20:37:

> Ich spreche weder vom Urheber dieses Geschwafels (dem ich kaum
> zutraue, alten von neuen Hüten unterscheiden zu können) noch von

alte

> dessen Perpetuierern. Sondern von einer effektiven, umkehrbaren
> Umrechnungsmöglichkeit zwischen Zahlen (inklusive rationalen) und
> endlichen Wörtern.

PS, falls das unklar gewesen sein sollte: mein Posting betraf
nicht die hiesige Tretmühle, sondern war ein Erklärungsversuch
zu deiner Klage über Beachtungsmangel.

Fritz Feldhase

unread,
Feb 11, 2024, 5:07:56 PMFeb 11
to
On Sunday, February 11, 2024 at 5:53:59 PM UTC+1, Fritz Feldhase wrote:

> W e l c h e natürliche Zahl ist denn im Schnitt über alle (unendlichen) Endsegmente enthalten, wenn es für j e d e natürliche Zahl einen (unendlichen) Endabschnitt gibt, in dem sie n i c h t enthalten ist?
>
> Oder anders: W i e k a n n eine natürliche Zahl im Schnitt über alle (unendlichen) Endsegmente enthalten sein, wenn es für j e d e natürliche Zahl einen (unendlichen) Endabschnitt gibt, in dem sie n i c h t enthalten ist?

Schulkinder würden vermutlich, anders als Sie Mückenheim, folgende anschauliche Erläuterung verstehen können:
______________________________________________

Wir können uns die natürlichen Zahlen als Elemente einer unendlichen Folge von Zahlen vorstellen:

1 2 3 4 5 ...

Die Endsegmente (bzw. deren Inhalte) könne wir dann durch "(" andeuten: Dabei sind alle "Zahlen", die rechts von "(" stehen, Elemente des durch diese Klammer angedeuten Endsegments und keine "Zahl" die links von "(" steht ist ein Element des durch diese Klammer angedeuten Endsegments.

Also so:

(1 (2 (3 (4 (5 ...

Nun steht JEDE natürliche "Zahl" offenbar links von einer Klammer (z. B. "1 (", "2 (", usw.)

Das bedeutet, dass diese "Zahl" nicht Element des Endsegments ist, das "unmittelbar rechts von ihr" durch "(" angedeutet ist.

Mit anderen Worten, JEDE natürliche Zahl ist in mindestens einem Endsegment NICHT als Element enthalten. Wieder anders: KEINE natürliche Zahl ist in ALLEN Endsegmenten als Element enthalten.

Da der Schnitt über alle Endsegmente genau jene natürlichen Zahlen enthält, die in ALLEN Endsegmenten als Elemente enthalten sind, ist der Schnitt über alle Endsegmente leer.

WM

unread,
Feb 12, 2024, 5:31:36 AMFeb 12
to
On 11.02.2024 16:59, Ralf Bader wrote:
> On 02/11/2024 12:10 PM, Ganzhinterseher wrote:
>>

>> Das ändert nichts daran, dass die Cantorsche Methode versagt:
>> XOOO...
>> XOOO...
>> XOOO...
>> XOOO...
>> ...
>> Hier kann man die X beliebig, nach Cantor oder anders, mit O
>> vertauschen, ohne dass ein O weniger resultiert. Damit ist Cantor
>> widerlegt. Dass andere Argumente anderes ergeben, ändert daran überhaupt
>> nichts. Es spricht allenfalls dafür, dass dunkle Zahlen existieren und
>> sich der näheren Analyse entziehen. Aber wie Fritsche und andere
>> Matheologen hier, das Argument einfach nicht zur Kenntnis zu nehmen,
>> zeugt von Fanatismus, Dogmatismus und vor allem unwissenschaftlichem
>> Benehmen.
>
> Das ist nicht "Cantors Methode",

Doch, es ist Cantors Methode, wobei statt der natürlichen Zahlen die
Ganzzahlbrüche als Indizes verwendet werden. Das kann aber nicht als
Begründung dafür dienen, dass es nicht Original Cantor sei. Denn die
Bijektion n <--> n/1 sollte in jedem Falle funktionieren.

> Natürlich verschwindet keine 0

Es geht um Os

> durch
> Vertauschen mit den X, sondern sie ändert nur ihren Platz.

So ist es. Und wer trotzdem an die Nummerierbarkeit alle Brüche glaubt,
verweigert offenbar das Denken.

Gruß, WM
>

WM

unread,
Feb 12, 2024, 5:35:22 AMFeb 12
to
Fritz Feldhase schrieb am Sonntag, 11. Februar 2024 um 17:53:59 UTC+1:
> On Sunday, February 11, 2024 at 11:50:07 AM UTC+1, WM wrote:

> W e l c h e natürliche Zahl ist denn im Schnitt über alle
(unendlichen) Endsegmente enthalten,

Wenn unendliche viele enthalten sind, so sind jedenfalls welche
enthalten. Da man sie nicht näher spezifizieren kann, sind sie dunkel.
Aber sie einfach wegzudenken, obwohl unendlich viele drin sind, ist falsch.

Gruß, WM

WM

unread,
Feb 12, 2024, 5:38:37 AMFeb 12
to
On 11.02.2024 20:18, Andreas Leitgeb wrote:
> WM <wolfgang.m...@outlook.de> wrote:

>> Wenn alles in Hexas dargestellt wird, dann gibt es keine überschüssigen.
>
> Jaja, aber was, wenn nicht?

Das ist eine Frage, die später zu erörtern ist.
>
> Hast du schon darüber nachgedacht, wie das möglich ist, dass die Menge
> der natürlichen Zahlen mit jeder Basis-darstellung "gleichzahlig" sein
> soll, wenn die Mengen der Basis-darstellungen untereinander für
> verschiedene Basen nicht "gleichzahlig" sind?

Die Mengen von natürlichen Zahlendarstellungen in verschiedenen Basen
sind offenbar verschieden groß.
>
> Und wie geht es dir beim Gedanken daran, dass durch geschickte Wahl der
> Darstellung die Menge der Bruch-darstellungen eine echte Teilmenge der
> Dezimalzahlen ist?

Das ist falsch, allenfalls ein Widerspruch, der unendliche
Größenvergleiche verbietet oder sinnlos macht.

Gruß, WM

Fritz Feldhase

unread,
Feb 12, 2024, 6:48:02 AMFeb 12
to
On Monday, February 12, 2024 at 11:35:22 AM UTC+1, WM wrote:
> Fritz Feldhase schrieb am Sonntag, 11. Februar 2024 um 17:53:59 UTC+1:
> > On Sunday, February 11, 2024 at 11:50:07 AM UTC+1, WM wrote:
> >
> > W e l c h e natürliche Zahl ist denn im Schnitt über alle (unendlichen) Endsegmente enthalten,
> > wenn es für j e d e natürliche Zahl einen (unendlichen) Endabschnitt gibt, in dem sie n i c h t
> > enthalten ist?

Keine Antwort. Stattdessen:

> Wenn unendliche viele enthalten sind, <blubber>

Es sind aber nicht unendlich viel enthalten, daher die Frage:

W i e k a n n eine natürliche Zahl im Schnitt über alle (unendlichen) Endsegmente enthalten sein,
wenn es für j e d e natürliche Zahl einen (unendlichen) Endabschnitt gibt, in dem sie n i c h t
enthalten ist?

Please elaborate.

Fritz Feldhase

unread,
Feb 12, 2024, 6:55:16 AMFeb 12
to
Zur Erinnerung:

"Der Schnitt [besteht aus dem], was in allen [(unendlichen) Endsegmenten] enthalten ist." (WM)

Die Frage lauter also eigentlich: Wie kann _etwas_ im Schnitt über alle (unendlichen) Endsegmente enthalten sein, wenn es _nichts_ gibt, das in allen (unendllichen) Endsegmenten enthalten ist

Please elaborate, Mückenheim!

WM

unread,
Feb 12, 2024, 9:37:07 AMFeb 12
to
Fritz Feldhase schrieb am Montag, 12. Februar 2024 um 12:48:02 UTC+1:
> On Monday, February 12, 2024 at 11:35:22 AM UTC+1, WM wrote:
> > Fritz Feldhase schrieb am Sonntag, 11. Februar 2024 um 17:53:59 UTC+1:
> > > On Sunday, February 11, 2024 at 11:50:07 AM UTC+1, WM wrote:
> > >
> > > W e l c h e natürliche Zahl ist denn im Schnitt über alle
(unendlichen) Endsegmente enthalten,
> > > wenn es für j e d e natürliche Zahl einen (unendlichen)
Endabschnitt gibt, in dem sie n i c h t
> > > enthalten ist?

> > Wenn unendliche viele enthalten sind, so sind jedenfalls welche
enthalten.

> Es sind aber nicht unendlich viel enthalten, daher die Frage:
> W i e k a n n eine natürliche Zahl im Schnitt über alle (unendlichen)
Endsegmente enthalten sein,
> wenn es für j e d e natürliche Zahl einen (unendlichen) Endabschnitt
gibt, in dem sie n i c h t
> enthalten ist?

Und wie können dann in jedem Endsegment unendlich viele Zahlen sein?
> Please elaborate.

Die Sache ist doch ganz offensichtlich.
Natürlich verschwinden im Laufe der Folge alle natürlichen Zahlen aus
den Endsegmenten, und zwar einzeln, eine nach der anderen, nicht
überstürzt und panikartig, denn es gilt
∀k ∈ ℕ : E(k+1) = E(k) \ {k}.
Wäre das nicht so, dann könnte man gar nicht daran denken, unendliche
Mengen zu nummerieren. Schon der Versuch wäre zum Scheitern verurteilt.
Denn man hätte nur den endlichen Anfangsabschnitt der unendlichen Menge
ℕ zur Verfügung.
Deine Behauptung, dass alle Endsegmente unendlich seien, macht allen
Cantorschen Bemühungen den Garaus. Das solltest Du eigentlich selbst merken.

Gruß, WM

WM

unread,
Feb 12, 2024, 9:42:59 AMFeb 12
to
Es gibt tatsächlich nichts, was in allen enthalten ist. Daher ist der
Schnitt über alle Endsegmente leer.

Aber es gibt sicher etwas, das in allen Endsegmenten, die etwas
enthalten, enthalten ist. Kannst Du das akzeptieren? In allen
Endsegmenten, die etwas enthalten, ist etwas enthalten. Das kann man
zwar nicht beschreiben, aber es ist nun einmal der gewöhnlichen Logik
nach der Fall. Der einzige Ausweg aus dieser paradoxen Situation
besteht darin, dunkle Zahlen zu akzeptieren.

Gruß, WM

Fritz Feldhase

unread,
Feb 12, 2024, 10:44:36 AMFeb 12
to
On Monday, February 12, 2024 at 3:37:07 PM UTC+1, WM wrote:
> Fritz Feldhase schrieb am Montag, 12. Februar 2024 um 12:48:02 UTC+1:
> >
> > W i e k a n n eine natürliche Zahl im Schnitt über alle (unendlichen) Endsegmente enthalten sein,
> > wenn es für j e d e natürliche Zahl einen (unendlichen) Endabschnitt gibt, in dem sie n i c h t
> > enthalten ist?

Keine Antwort, stattdessen - wie bei kleinen Kindern - eine "Gegenfrage":

> Und wie können dann in jedem Endsegment unendlich viele Zahlen sein?

Das eine hat mit dem anderen nichts zu tun.*) Ich habe Dir das doch schon erklärt.

DIE FRAGE IST:

W i e k a n n eine natürliche Zahl im Schnitt über alle (unendlichen) Endsegmente enthalten sein, wenn es für j e d e natürliche Zahl einen (unendlichen) Endabschnitt gibt, in dem sie n i c h t enthalten ist?

> Die Sache ist doch ganz offensichtlich.

Das klingt vielversprechend. Kommt jetzt die Antort auf meine einfache/klare Frage?

> Natürlich verschwinden im Laufe der Folge alle natürlichen Zahlen aus
> den Endsegmenten, und zwar einzeln, eine nach der anderen,

Das ist in der Tat so, denn es gilt: An e IN: n !e E(n+1).

D. h. FÜR JEDE natürliche Zahl gibt es ein (unendliches) Endsegment, in dem sie nicht enthalten ist. (Früher oder später verschwindet also j e d e natürliche Zahl aus den (unendlichen) Endsegmenten; wenn es auch kein Folgenglied gibt, aus dem alle natürlichen Zahlen "verschwunden" sind. Denn _für jedes n e IN_ enthält E(n+1) noch {n+1, n+2, n+3, ...}.)

> nicht überstürzt und panikartig,

Hat das jemals jemand (außer Dir) behauptet? :-)

> denn es gilt ∀k ∈ ℕ : E(k+1) = E(k) \ {k}.

Genau.

> Wäre das nicht so,

Es ist aber so. Wir brauchen das daher hier nicht weiter zu vertiefen. Was mich MEHR interessieren würde, ist daher die Antwort auf die Frage:

| W i e k a n n eine natürliche Zahl im Schnitt über alle (unendlichen) Endsegmente enthalten sein, wenn es für j e d e natürliche Zahl einen (unendlichen) Endabschnitt gibt, in dem sie n i c h t enthalten ist?

Insbesondere wo doch gilt:

| "Der Schnitt [besteht aus dem], was in allen [(unendlichen) Endsegmenten] enthalten ist." (WM)

Die Frage lauter also eigentlich:

| Wie kann _etwas_ im Schnitt über alle (unendlichen) Endsegmente enthalten sein, wenn es _nichts_ gibt, das in allen (unendllichen) Endsegmenten enthalten ist

Aha, da kommt doch noch was:

> Es gibt tatsächlich nichts, was in allen enthalten ist. Daher ist der Schnitt über alle Endsegmente leer.

So ist es.

Anmerkung: Alle Ensegmente sind unendlich, daher ist es hirnrissig ständig "unendliche Endsegmente" zu schreiben.

Nochmal: Alle Endsegmente sind unendlich, daher gilt: Ax(x ist ein Endsegment <-> x ist ein unendliches Endsegment).

Dein Bestreben ständig zwischen "Endsegmenten" und "unendlichen Endsegmenten" unterscheiden zu wollen, ist wohl ein Ausdruck Deiner geistig-mentalen Divergenz.

> [Die] Behauptung, dass alle Endsegmente unendlich seien, macht allen Cantorschen Bemühungen den Garaus.

Kann man noch dümmeres Zeug faseln, Mückenheim? Schwerlich.

Lassen wir den guten Cantor mal in Frieden ruhen und beschäftigen uns kurz mit Mengenlehre.

=============================================================================

Vielleicht ist Dir bekannt, dass die Menge der natürlichen Zahlen UNENDLICH ist, Mückenheim.

Man kann z. B. /unendlich/ nach Dedekind definieren (oder anders) ebenso die Menge IN in einer der üblichen Weisen. Es gilt dann:

(1) IN ist unendlich.

Nun kann man die sogenannten Endsegmente sehr bequem SO definieren, dass man ERST die Anfangsabschnitte A(n) := {m e IN : m < n} definiert und dann E(n) (mit n e IN) - also die Endsegmente - wie folgt:

(2) E(n) = IN \ A(n) (n e IN)

Hinweis: So hat das auch der gute Cantor mit seinen Abschnitten und Resten gemacht.

Nun kann man leicht zeigen, dass die Anfangsabschnitte allesamt endlich sind, dass also gilt:

(3) An e IN: A(n) ist endlich.

Daraus ergibt sich dann (aus (1), (2) und (3)) leicht, dass alle Endsegmente unendlich sind:

(4) An e IN: E(n) ist unendlich.

(Der einfache Beweis sei dem Leser als Übung überlassen).

Fritz Feldhase

unread,
Feb 12, 2024, 11:10:34 AMFeb 12
to
On Monday, February 12, 2024 at 3:42:59 PM UTC+1, WM wrote:
> Fritz Feldhase schrieb am Montag, 12. Februar 2024 um 12:55:16 UTC+1:
> > On Monday, February 12, 2024 at 12:48:02 PM UTC+1, Fritz Feldhase wrote:
> >
> > "Der Schnitt [besteht aus dem], was in allen [(unendlichen) Endsegmenten] enthalten ist." (WM)
> >
> > Die Frage lauter also eigentlich: Wie kann _etwas_ im Schnitt über alle (unendlichen) Endsegmente enthalten sein, wenn es _nichts_ gibt, das in allen (unendllichen) Endsegmenten enthalten ist
> >
> Es gibt tatsächlich nichts, was in allen enthalten ist. Daher ist der Schnitt über alle Endsegmente leer.

Sehr schön. So ist es.

> Aber es gibt sicher etwas, das in allen Endsegmenten [...] enthalten ist.

Siehst Du nicht, dass das in DIREKTEM Widerspruch steht zu dem, was Du gerade eben oben gesagt hast?

<facepalm>

(a) "Es gibt nichts, was in allen Ensegmenten enthalten ist." (WM)
(b) "Aber es gibt sicher etwas, das in allen Endsegmenten enthalten ist." (WM)

Du hast wirklich ein Rad am, Mann.

==============================================================

Also nochmal, WAS GILT, ist:

(1) AE e ENDSEG: En e IN: n e E.
"Für jedes Endsegment gibt es eine natürliche Zahl, die in ihm enthalten ist."

(Das impliziert, dass kein Endsegment leer ist: AE e ENDSEG: E =/= { }.)

Was NICHT GILT, ist:

(2) En e IN: AE e ENDSEG: n e IN.
"Es gibt eine natürliche Zahl, die in allen Endsegmenten enthalten ist"

Denn "es gibt tatsächlich nichts, was in allen [Endsegmenten] enthalten ist" (WM).

Merke: Der "quantifier shift" ist aus gutem Grund keine erlaubte Schlussweise. (Denn mittels eines "quantifier shifts" (2) aus (1) "zu folgern", wäre ein Fehlschluss.)

Mit anderen Worten, es GILT vielmehr:

(3) ~En e IN: AE e ENDSEG: n e IN.
"Es gibt KEINE natürliche Zahl, die in allen Ensegmenten enthalten ist."

> [In] Endsegmenten, die etwas enthalten, ist etwas enthalten.

Ja, das ist in der Tat so.

In der Mengenlehre gilt ganz allgemein:

Ax(x =/= { } -> Ey(y e x)).
"Jede Menge, die nicht gleich der leeren Menge ist, enthält 'etwas'." :-)

> Das kann man zwar nicht beschreiben,

Bei den Endsegmenten [und vielen anderen Mengen] kann man das SEHR GUT beschreiben, Mückenheim.

Hinweis: Das Endsegment E(n) enthält die unendlich vielen (natürlichen) Zahlen n+1, n+2, n+3, ... [für jede natürlich Zahl n].

Man kann das sogar dann SEHR GUT beschreiben, wenn der Index des Endsegments, um das es geht, nicht bekannt/gegeben ist.

Sei E ein Endsegment, dann enthält E die folgenden [unendlich vielen] Elemente: min(E), min(E)+1, min(E)+2, min(E)+3, ..., wobei min(E), min(E)+1, min(E)+2, min(E)+3, ... natürliche Zahlen sind. (Hinweis: Jede Teilmenge der Menge der natürlichen Zahlen enthält ein klenstes Element.)

Das ist nach

> der gewöhnlichen Logik nach der Fall.

Gut, dass wir darüber geredet haben, Mückenheim.

Fritz Feldhase

unread,
Feb 12, 2024, 11:18:41 AMFeb 12
to
On Monday, February 12, 2024 at 3:37:07 PM UTC+1, WM wrote:

Weil, so schön war, hier nochmal.

> dass alle Endsegmente unendlich [sind]

lässt sich [im Kontext der Mengenlehre] leicht zeigen/beweisen. Der Beweis beruht natürlich auf ein paar Grundannahmen (die man im Kontext der Mengenlehre leicht beweisen kann).

Bekanntlich ist (im Kontext der Mengenlehre) die Menge der natürlichen Zahlen unendlich.

Man kann z. B. /unendlich/ nach Dedekind definieren (oder anders) ebenso die Menge IN in einer der üblichen Weisen. Es gilt dann:

(1) IN ist unendlich.

Nun kann man die sogenannten Endsegmente sehr bequem SO definieren, dass man ERST die Anfangsabschnitte A(n) := {m e IN : m < n} (n e IN) definiert und dann E(n) (mit n e IN) - also die Endsegmente - wie folgt:

(2) E(n) = IN \ A(n) (n e IN).

(Hinweis: So hat das auch der gute Cantor mit seinen Abschnitten und Resten gemacht.)

Nun kann man leicht zeigen, dass die Anfangsabschnitte allesamt endlich sind, dass also gilt:

(3) An e IN: A(n) ist endlich.

Daraus ergibt sich dann (aus (1), (2) und (3)) leicht, dass alle Endsegmente unendlich sind:

(4) An e IN: E(n) ist unendlich.

qed (Der einfache Beweis sei dem Leser als Übung überlassen.)

Ralf Bader

unread,
Feb 12, 2024, 11:39:15 AMFeb 12
to
On 02/12/2024 11:31 AM, WM wrote:
> On 11.02.2024 16:59, Ralf Bader wrote:
>> On 02/11/2024 12:10 PM, Ganzhinterseher wrote:
>>>
>
>>> Das ändert nichts daran, dass die Cantorsche Methode versagt:
>>> XOOO...
>>> XOOO...
>>> XOOO...
>>> XOOO...
>>> ...
>>> Hier kann man die X beliebig, nach Cantor oder anders, mit O
>>> vertauschen, ohne dass ein O weniger resultiert. Damit ist Cantor
>>> widerlegt. Dass andere Argumente anderes ergeben, ändert daran überhaupt
>>> nichts. Es spricht allenfalls dafür, dass dunkle Zahlen existieren und
>>> sich der näheren Analyse entziehen. Aber wie Fritsche und andere
>>> Matheologen hier, das Argument einfach nicht zur Kenntnis zu nehmen,
>>> zeugt von Fanatismus, Dogmatismus und vor allem unwissenschaftlichem
>>> Benehmen.
>>
>> Das ist nicht "Cantors Methode",
>
> Doch, es ist Cantors Methode,

Nein, ist es nicht und wird es auch nicht durch Ihr saublödes Geschwafel.

Fritz Feldhase

unread,
Feb 12, 2024, 12:21:21 PMFeb 12
to
On Monday, February 12, 2024 at 4:44:36 PM UTC+1, Fritz Feldhase wrote:
> On Monday, February 12, 2024 at 3:37:07 PM UTC+1, WM wrote:

Ein Nachtrag:

> > Und wie können dann in jedem Endsegment unendlich viele [natürliche] Zahlen sein?
> >
> Das eine hat mit dem anderen nichts zu tun. [...]

Dass es keine natürliche Zahl gibt, die in allen Endsegment enthalten ist (und damit auch nicht (ein und dieselben) unendlich viele(n) natürliche(n) Zahlen, die in allen Endsegmenten enthalten sind), schließt natürlich nicht aus, dass es in jedem Endsegment unendlich viele natürliche Zahlen gibt.

Dieser Umstand wäre NUR DANN ein "Problem", wenn der "quantifier shift" eine logisch gültige Schlussweise wäre (dem ist aber nicht so).

...

Eines Deiner Probleme scheint zu sein, dass Du "geistig-mental" nicht zwischen

AE e ENDSEG: E^oo n e IN: n e E (wahr)
"Zu jedem Endsegment gibt es unendlich viele natürliche Zahlen, die in ihm enthalten sind."

und

E^oo n e IN: AE e ENDSEG: n e E (falsch)
"Es gibt unendlich viele natürliche Zahlen, die in jedem Endsegment enthalten sind."*)

unterscheiden kannst. (Vermutlich ist Dir WEDER der mathematische Sachverhalt, NOCH der korrekte Gebrauch von Quantoren klar.)

*) Das muss vielleicht - extra für Dich - noch erklärt werden. Was hier ausgesagt wird, ist, dass es bestimmte/gewisse natürliche Zahlen gibt, und zwar unendlich viele von ihnen, die in allen Endsegmenten enthalten sind. Das kann man äquivalent auch so aussagen: Es gibt eine unendliche Teilmenge der Menge der natürlichen Zahlen für die gilt, dass sie Teilmenge jeden Endsegments ist: EM c IN: M ist unendlich & AE e ENDSEG: M c E. [Das wäre(n) dann Deine Kernmenge(n). Diese gibt es aber nicht.]

_______________________________________________________________________


Dass sich die beiden Sachverhalte "nicht grundsätzich" ausschließen, macht man sich viell. am Besten MIT ANDEREN MENGEN als den Endsegmenten klar.

Statt den Endsegmenten E(n) wollen wir hier einmal die Mengen P(n) ["Primpotenzmengen"] betrachten, die wie folgt definiert sind:

P(n) = {p_n, (p_n)^2, (p_n)^3, ...} wo p_n die n-te Primzahl (der der größe nach geordneten Primzahlen) sein soll.

Also: P(1) = {2, 4, 8, ...}, P(2) = {3, 9, 27, ...}, P(3) = {5, 25, 125, ...}, usw.

Die Menge all dieser Primpotenzmengen sein PP. Also PP := {P(n) : n e IN}.

Es gilt dann offensichtlich:

(1) AP e PP: E^oo n e IN: n e P ,
"Zu jeder Primpotenzmenge gibt es unendlich viele natürliche Zahlen, die in ihr enthalten sind."

aber NICHT:

(2) E^oo n e IN: AP e PP: n e E .
"Es gibt unendlich viele natürliche Zahlen, die in jeder Primpotenzmenge enthalten sind."

Denn es gibt nicht einmal EINE natürliche Zahlen, die in jeder Primpotenzmenge enthalten sind. [Es ist klar, dass die Primpotenzmengen "paarweise dijunkt" sind, dass also für alle n,m e IN mit n =/= m gilt: P(n) n P(m) = { }.]

Einmal mehr zeigt dieser Umstand, dass der "quantifier shift" ein FEHLSCHLUSS ist.

"_Quantifier shift fallacy_

A fallacy of reversing the order of two quantifiers. The common form is that of moving from a statement of the form ‘every x has a related y’ to one of the form ‘there is some y related to every x’. An easily detected instance would be inferring from ‘everyone has a mother’ to ‘there is someone who is everyone's mother’. More subtle instances would be trading on the two different meanings that can be given to statements like ‘there is some proposition presupposed in every investigation’. Does this mean that for all investigations there is some possibly different presupposition, or that there is some unique common presupposition? Similarly ‘there is something that is the meaning of all our activities’, which smooths the fallacious transition from ‘each activity has a meaning’ which is probably true to ‘there is a Purpose common to all of them’ which is probably false. The fallacy is obvious in quantification theory, where it is represented as moving from (∀x)(∃y)(Rxy) to (∃y)(∀x)(Rxy)."

Quelle: https://www.oxfordreference.com/display/10.1093/oi/authority.20110803100357607

Fritz Feldhase

unread,
Feb 12, 2024, 12:43:25 PMFeb 12
to
On Monday, February 12, 2024 at 11:31:36 AM UTC+1, WM wrote:
> On 11.02.2024 16:59, Ralf Bader wrote:
> > On 02/11/2024 12:10 PM, Ganzhinterseher wrote:
> > >
> > > Das ändert nichts daran, dass die Cantorsche Methode versagt:
> > >
> > > XOOO...
> > > XOOO...
> > > XOOO...
> > > XOOO...
> > > ...
> > > Hier kann man die X beliebig [per Paarvertauschung(en)] mit O
> > > vertauschen, ohne dass ein O weniger resultiert. Damit ist Cantor
> > > widerlegt. [...]
> > >
> > Das ist nicht "Cantors Methode",
> >
> Doch, es ist Cantors Methode

Ach, ist das so? Wo genau hat Cantor eine Folge von (unendlichen) Matrizen betrachtet und behauptet, dass die Existenz dieser Folge die Abzählbarkeit der Brüche impliziert?

Bitte um ein Zitat.

Hinweis: Du bringst da etwas _Grundlegendes_ durcheinander. :-)

Cantor hat eine Funktion (also eine Bijektion) angegeben, die zeigt/beweist, dass die Element der "Matrix" (wenn man denn hier, anders als Cantor es getan hat, von einer Matrix sprechen will)

1/1 1/2 1/3 ...
2/1 2/2 2/3 ...
3/1 3/2 3/3 ...
...

abzählbar sind. DAS ist etwas gänzlich anderes als das, was Du da aufführts, Mückenheim.

Näheres dazu findest Du hier: https://de.wikipedia.org/wiki/Cantorsche_Paarungsfunktion

"Cantors Methode" (die er in diesem Zusammenhang angewandt hat) ist hier erklärt: https://de.wikipedia.org/wiki/Cantors_erstes_Diagonalargument

Dein saudummer Scheißdreck, den Du in diesem Zusammenhang daherlaberst, hat nichts mit "Cantors Methode" zu tun.

WM

unread,
Feb 13, 2024, 3:30:35 AMFeb 13
to
On 10.02.2024 22:24, Andreas Leitgeb wrote:
> Ganzhinterseher <wolfgang.m...@tha.de> wrote:
>> Es geht bei der Messung von Zahldarstellungen um Zahlendarstellungen.
>> Es gibt ja auch viel mehr Brüche als rationale Zahlen.
>
> Aber wie ist das nun, wenn man den dezimal-darstellungen der natürlichen
> Zahlen noch jene hexadezimalen Zahlen, die nicht auch dezimal sind, zur
> Seite stellt? Reicht das dann vielleicht schon für die Brüche?
>
Ja, das ist wohl so. Jedenfalls reichen Binärzahlen für alle in
Unärzahlen darstellbaren Brüche.
Beweis: Es gibt unendlich viele disjunkte Folgen von Unärdarstellungen
natürlicher Zahlen in der Menge der Binärzahlen, z. B.
0, 00, 000, ...
1,11, 111, ...
10, 100, 1000, ...
110, 1100, 11000, ...
1110, 11100, 111000, ...
und so weiter.

Ich schlage deshalb vor, das Symbol ℕ nur noch für die Menge der
Unärdarstllungen der natürlichen Zahlen zu verwenden (denn die ist die
natürlichste Darstellungsform), womit denn alle oben angegebenen Folgen
die Anzahl |ℕ| hätten.

Für die potentiell unendlichen Kollektionen der erkennbaren
Zahlendarstellungen besteht natürliche kein Unterschied. Zwischen diesen
Mengen funktionieren Cantors Bijektionen nach wie vor.

Gruß, WM

WM

unread,
Feb 13, 2024, 3:53:44 AMFeb 13
to
On 11.02.2024 08:00, Ralf Bader wrote:

> Man kann die mit 1 beginnenden Wörter endlicher Länge über dem Alphabet
> {0,1} auffassen einerseits als Binärdarstellungen natürlicher Zahlen,
> andererseits aber auch als Darstellungen rationaler Zahlen bzw. Brüchen,
> wobei die letzte 1 in so einem Wort die Rolle des Bruchstrichs
> übernimmt, der Zähler ist in Binärdarstellung durch das Teilwort links
> dieser 1 gegeben, der Nenner durch die Anzahl der auf diese 1 folgenden
> Nullen. Randfälle, wenn etwa die letzte 1 das Wort beendet, können
> relativ zwanglos separat verarztet und einsortiert werden.

Du gibst eine gemischt binär-unäre Darstellungsform an. In der Tat kann
man die Matrix aller unär dargestellten Brüche mit Binärzahlen
indizieren. Für die Widerlegung von Cantors Bijektion ist aber in beiden
Fällen dieselbe Darstellungsform zu wählen, denn die hat Cantor
automatisch mit dezimal zugrundegelegt. Der durch die Darstellungsform
bedingte Unterschied in der Anzahl wurde bisher überhaupt nicht beachtet.

Anmerkung: Alle Folgen "bis zur letzten 1" sind durch die Anzahl |ℕ|
beschränkt. Durch Binärdarstellung sind allerdings unendlich mehr
Zahldarstellungen möglich.

Gruß, WM

WM

unread,
Feb 13, 2024, 3:56:56 AMFeb 13
to
On 12.02.2024 17:39, Ralf Bader wrote:
> On 02/12/2024 11:31 AM, WM wrote:
>> On 11.02.2024 16:59, Ralf Bader wrote:
>>> On 02/11/2024 12:10 PM, Ganzhinterseher wrote:
>>>>
>>
>>>> Das ändert nichts daran, dass die Cantorsche Methode versagt:
>>>> XOOO...
>>>> XOOO...
>>>> XOOO...
>>>> XOOO...
>>>> ...
>>>> Hier kann man die X beliebig, nach Cantor oder anders, mit O
>>>> vertauschen, ohne dass ein O weniger resultiert. Damit ist Cantor
>>>> widerlegt. Dass andere Argumente anderes ergeben, ändert daran
>>>> überhaupt
>>>> nichts. Es spricht allenfalls dafür, dass dunkle Zahlen existieren und
>>>> sich der näheren Analyse entziehen. Aber wie Fritsche und andere
>>>> Matheologen hier das Argument einfach nicht zur Kenntnis zu nehmen,
>>>> zeugt von Fanatismus, Dogmatismus und vor allem unwissenschaftlichem
>>>> Benehmen.
>>>
>>> Das ist nicht "Cantors Methode",
>>
>> Doch, es ist Cantors Methode,
>
> Nein,

Du lügst und Du weißt das auch. Andernfalls würdest Du Deine "besseren"
Erkenntnisse natürlich vortragen.

Gruß, WM

WM

unread,
Feb 13, 2024, 4:02:26 AMFeb 13
to
Fritz Feldhase schrieb am Montag, 12. Februar 2024 um 18:43:25 UTC+1:
> On Monday, February 12, 2024 at 11:31:36 AM UTC+1, WM wrote:
> > On 11.02.2024 16:59, Ralf Bader wrote:
> > > On 02/11/2024 12:10 PM, Ganzhinterseher wrote:
> > > >
> > > > Das ändert nichts daran, dass die Cantorsche Methode versagt:
> > > >
> > > > XOOO...
> > > > XOOO...
> > > > XOOO...
> > > > XOOO...
> > > > ...
> > > > Hier kann man die X beliebig [per Paarvertauschung(en)] mit O
> > > > vertauschen, ohne dass ein O weniger resultiert. Damit ist Cantor
> > > > widerlegt. [...]
> > > >
> > > Das ist nicht "Cantors Methode",
> > >
> > Doch, es ist Cantors Methode
> Ach, ist das so? Wo genau hat Cantor eine Folge von (unendlichen)
Matrizen betrachtet und behauptet, dass die Existenz dieser Folge die
Abzählbarkeit der Brüche impliziert?

Es geht nicht u die Sprache, sondern u die Methode.

> Cantor hat eine Funktion (also eine Bijektion) angegeben, die
zeigt/beweist, dass die Element der "Matrix" (wenn man denn hier, anders
als Cantor es getan hat, von einer Matrix sprechen will)
>
> 1/1 1/2 1/3 ...
> 2/1 2/2 2/3 ...
> 3/1 3/2 3/3 ...
> ...

Ja, das will und darf man, denn es ändert nichts am Sachverhalt.
>
> abzählbar sind. DAS ist etwas gänzlich anderes als das, was Du da
aufführts, Mückenheim.

Nein, es ist genau dasselbe. Cantor hat lediglich vergessen, seine
Mittel genauer zu untersuchen.

Es ist erbärmlich und geradezu krankhaft, die Verwendung der
Cantorschzen Funktion
k = (m + n - 1)(m + n - 2)/2 + m
und der resultierenden Cantorschen Folge
1/1, 1/2, 2/1, 1/3, 2/2, 3/1, 1/4, 2/3, 3/2, 4/1, 1/5, 2/4, 3/3, 4/2,
5/1, 1/6, 2/5, 3/4, 4/3, 5/2, 6/1, ...
in Form von Matrizen als unzulässig zu deklarieren.

Gruß, WM


Gruß, WM

Rainer Rosenthal

unread,
Feb 13, 2024, 4:43:49 AMFeb 13
to
Am 13.02.2024 um 09:53 schrieb WM:

> Der durch die Darstellungsform bedingte Unterschied
> in der Anzahl wurde bisher überhaupt nicht beachtet.

Das liegt daran, dass niemand eine von der Darstellungsform abhängige
Anzahl braucht. Mit abstraktem Denken hast Du es halt so gar nicht und
staunst immer noch darüber, dass 3 Äpfel genau so viele sind wie 3 Birnen.

Die dahinter steckende Idee der Bijektion hast Du nicht verstanden.

Die einzige ehrliche Aussage in diesem Zusammenhang war, dass Du die
mathematischen Anfängervorlesungen geschwänzt hast, weil Du zum Lehren
und nicht zum Lernen geboren wurdest.

Gruß,
RR



Stefan Schmitz

unread,
Feb 13, 2024, 5:34:20 AMFeb 13
to
Am 13.02.2024 um 10:43 schrieb Rainer Rosenthal:

> Die einzige ehrliche Aussage in diesem Zusammenhang war, dass Du die
> mathematischen Anfängervorlesungen geschwänzt hast, weil Du zum Lehren
> und nicht zum Lernen geboren wurdest.

Hat er das wirklich mal so gesagt?
Man kann ja schlecht etwas lehren, was man selbst gar nicht gelernt hat.

WM

unread,
Feb 13, 2024, 5:46:37 AMFeb 13
to
Stefan Schmitz schrieb am Dienstag, 13. Februar 2024 um 11:34:20 UTC+1:
> Am 13.02.2024 um 10:43 schrieb Rainer Rosenthal:
>
> > Die einzige ehrliche Aussage in diesem Zusammenhang war, dass Du die
> > mathematischen Anfängervorlesungen geschwänzt hast, weil Du zum Lehren
> > und nicht zum Lernen geboren wurdest.
> Hat er das wirklich mal so gesagt?

Natürlich nicht. Rosenthal ist ein Lügner vom Scheitel bis zur Sohle.

Gruß, WM

WM

unread,
Feb 13, 2024, 5:54:53 AMFeb 13
to
On 11.02.2024 20:18, Andreas Leitgeb wrote:
> WM <wolfgang.m...@outlook.de> wrote:

> Und wie geht es dir beim Gedanken daran, dass durch geschickte Wahl der
> Darstellung die Menge der Bruch-darstellungen eine echte Teilmenge der
> Dezimalzahlen ist?
Das ist nicht der Fall - in keiner gemeinsamen Basis.

Gruß, WM
>

Jens Kallup

unread,
Feb 13, 2024, 7:13:47 AMFeb 13
to
Am 2024-02-13 um 09:30 schrieb WM:
> Ja, das ist wohl so. Jedenfalls reichen Binärzahlen für alle in
> Unärzahlen darstellbaren Brüche.
> Beweis: Es gibt unendlich viele disjunkte Folgen von Unärdarstellungen
> natürlicher Zahlen in der Menge der Binärzahlen, z. B.
> 0, 00, 000, ...
> 1,11, 111, ...
> 10, 100, 1000, ...
> 110, 1100, 11000, ...
> 1110, 11100, 111000, ...
> und so weiter.

stopp hier, und nicht weiter !
Für Deine spielerrei (auch mit der Matrix) ist hier kein Platz für oo
viele Folgen von:

E2( E1( n | m ) e IN ): Binärsystem

- E1, und E2 für den Quantor 1 und 2
- IN natürliche mathematische Objekte
- e für "Element von"
- n := 0.
- m := 1.
- or, oder: | für ODER

für Deine Matrix-Spielerrei wären dann 4 Bit das höchste
0000 = 1
1111 = F => 16

Es_n( i | j )
Er_n( i | j )

T_y_x ( Es_n, Er_n)

( (T_1_1, T_2_1, T_3_1, T_4_1) )
( )
( (T_1_2, T_2_2, T_3_2, T_4_2) )
M := ( )
( (T_1_3, T_2_3, T_3_3, T_4_3) )
( )
( (T_1_4, T_2_4, T_3_4, T_4_4) )


T_1_1 => 0000 => 0 => 1
T_4_4 => 1111 => F => 16

oder:

( 0, 1, 2, 3 )
( )
( 4, 5, 6, 7 )
M := ( )
( 8, 9, A, B )
( )
( C, D, E, F )

Du hast also maximal:

- 15 nullen
- 15 einsen

- 15 Varianten für 0 und 1 (die für 1 bis 16 stehen)
- eine "endliche" Anzahl Möglichkeiten, wie Du 0..F anordnen kannst

> Ich schlage deshalb vor, das Symbol ℕ nur noch für die Menge der
> Unärdarstllungen der natürlichen Zahlen zu verwenden (denn die ist die
> natürlichste Darstellungsform), womit denn alle oben angegebenen Folgen
> die Anzahl |ℕ| hätten.

das wäre Fatal.
IN hat sich für die Menge der natürliche Zahlen etabliert.
Dein Vorschlag ist wirr und irreführend.

JVR

unread,
Feb 13, 2024, 7:52:00 AMFeb 13
to
Nein, Mücke, Rosenthal ist kein Lügner. Wenn er Ihnen widerspricht, dann
hat er fast immer recht; und wenn er einen Fehler einsieht, dann korrigiert er
ihn.
Sie, andererseits, korrigieren Ihre Fehler niemals, aber ich halte Sie trotzdem
nicht für einen Lügner, d.h. für jemanden, der bei klarem Verstand
Aussagen macht, von denen er weiß, dass sie falsch sind.

Ralf Goertz

unread,
Feb 13, 2024, 8:22:02 AMFeb 13
to
Am Tue, 13 Feb 2024 11:46:35 +0100
schrieb WM <wolfgang.m...@tha.de>:
So wie du, als du behauptet hast, du würdest deine Newsgroup-Nachrichten
nicht auch als private Emails verschicken?

Fritz Feldhase

unread,
Feb 13, 2024, 8:22:41 AMFeb 13
to
On Tuesday, February 13, 2024 at 11:54:53 AM UTC+1, WM wrote:
> On 11.02.2024 20:18, Andreas Leitgeb wrote:
> >
> > Und wie geht es dir beim Gedanken daran, dass durch geschickte Wahl der
> > Darstellung die Menge der Bruch-Darstellungen eine echte Teilmenge der
> > Dezimalzahlen ist?
> >
> Das ist nicht der Fall

RBs Beitrag nicht gelsen oder nur nicht verstanden? [Von mir leicht modifiziert/angepasst:]

"Man kann die mit 1 beginnenden Wörter endlicher Länge über dem Alphabet
{0,1} auffassen einerseits als [Dezimaldarstellungen] natürlicher Zahlen,
andererseits aber auch als Darstellungen [der] Brüche[,]
wobei die letzte 1 in so einem Wort die Rolle des Bruchstrichs
übernimmt, der Zähler ist in Binärdarstellung durch das Teilwort links
dieser 1 gegeben, der Nenner durch die Anzahl der auf diese 1 folgenden
Nullen. Randfälle, wenn etwa die letzte 1 das Wort beendet, können relativ
zwanglos separat verarztet und einsortiert werden."*)

Man kann also die folgenden Dezimalzahlen:

110, 1100, 1010, 11000, ...

auffassen als Darstellungen der Brüche:

1/1, 1/2, 2/1, 1/3, ...

Wobei offensichtlich die hierfür benötigten Dezimalzahlen lediglich eine echte Teilmenge der (also aller) Dezimalzahlen ist.

____________________________________________

*) Vorschlag: Von vornherein nur die Wörter endlicher Länge über dem Alphabe {0,1} betrachten, die mit "1" beginnen und mit "0" enden sowie mind. zwei "1"en enthalten. Die letzte "1" (vor den darauf folgenden "0"en) übernimmt - wie eben schon erwähnt - die Rolle des Bruchstrichs. Es stehen dann nur noch "0"en nach dieser "1"; und zwar "minimal" eine (die "0" am Schluss), so dass der "minimale" Nenner >= 1 (dez) ist. Vor der letzten "1" steht "minimal" eine "1" oder eine Ziffernfolge (der Länge >= 2 (dez) mit Ziffern aus {"0", "1"}, die mit einer "1" beginnt, so dass auch der "minimale" Zähler >= 1 (dez) ist.

Fritz Feldhase

unread,
Feb 13, 2024, 8:30:52 AMFeb 13
to
On Tuesday, February 13, 2024 at 2:22:41 PM UTC+1, Fritz Feldhase wrote:

Kleine Korrektur ("endlich viele" und "endliche" ergänzt):

*) Vorschlag: Von vornherein nur die Wörter endlicher Länge über dem Alphabe {0,1} betrachten, die mit "1" beginnen und mit "0" enden sowie mind. zwei "1"en enthalten. Die letzte "1" (vor den darauf folgenden "0"en) übernimmt - wie eben schon erwähnt - die Rolle des Bruchstrichs. Es stehen dann nur noch (endlich viele) "0"en nach dieser "1"; und zwar "minimal" eine (die "0" am Schluss), so dass der "minimale" Nenner >= 1 (dez) ist. Vor der letzten "1" steht "minimal" eine "1" oder eine (endliche) Ziffernfolge (der Länge >= 2 (dez) mit Ziffern aus {"0", "1"}, die mit einer "1" beginnt, so dass auch der "minimale" Zähler >= 1 (dez) ist.

Damit ist gewährleistet, dass jedes solche Wort einen (positiven) Bruch darstellt. (Dass jeder (positive) Bruch auf diese Weise dargestellt werden kann, ist klar.)

Fritz Feldhase

unread,
Feb 13, 2024, 9:04:20 AMFeb 13
to
On Tuesday, February 13, 2024 at 10:43:49 AM UTC+1, Rainer Rosenthal wrote:

> Die einzige ehrliche Aussage in diesem Zusammenhang war, dass Du die
> mathematischen Anfängervorlesungen geschwänzt hast, weil Du zum Lehren
> und nicht zum Lernen geboren wurdest.

Ganz so hat er es dann wohl doch nicht gesagt, oder? :-)

Ist hier viell. Deine Phantasie etwas mit Dir durchgegangen?

Jedenfalls wissen wir, dass Herr Mückenheim gewisse mathematische Anfängervorlesungen (aus welchen Gründen auch immer) nicht gehört hat.

Man gewinnt gelegentlich den Eindruck, dass sich das äußerst negativ ausgewirkt hat. Und ja, AUSGERECHNET SO JEMAND fühlt sich berufen, ein "Mathelehrbuch" zu schreiben ... (and it shows)!

WM

unread,
Feb 13, 2024, 10:53:49 AMFeb 13
to
On 13.02.2024 13:51, JVR wrote:
> On Tuesday, February 13, 2024 at 11:46:37 AM UTC+1, WM wrote:

> Nein, Rosenthal ist kein Lügner.

Doch er ist ein Lügner, aber Du bist es auch, und dabei ein Neidhammel,
der seine Komplexe nur durch Beleidigungsversuche kompensieren kann. Was
mich aber kalt lässt. Schließlich ist Deine Person für mich und wohl die
meisten irrelevant.

> Sie, andererseits, korrigieren Ihre Fehler niemals,

Das ist schon wieder gelogen. Aber vermutlich sprichst Du Erkenntnisse
an, die Du überhaupt nicht verstehen kannst und denen Du deshalb misstraust.

Gruß, WM

WM

unread,
Feb 13, 2024, 10:54:59 AMFeb 13
to Fritz Feldhase
On 13.02.2024 14:22, Fritz Feldhase wrote:
> On Tuesday, February 13, 2024 at 11:54:53 AM UTC+1, WM wrote:
>> On 11.02.2024 20:18, Andreas Leitgeb wrote:
>>>
>>> Und wie geht es dir beim Gedanken daran, dass durch geschickte Wahl der
>>> Darstellung die Menge der Bruch-Darstellungen eine echte Teilmenge der
>>> Dezimalzahlen ist?
>>>
>> Das ist nicht der Fall
>
> RBs Beitrag nicht gelsen oder nur nicht verstanden?

Längst beantwortet und erklärt.

Gruß, WM

JVR

unread,
Feb 13, 2024, 11:23:57 AMFeb 13
to
Sie, andererseits, korrigieren Ihre Fehler niemals, aber ich halte Sie trotzdem
nicht für einen Lügner, d.h. für jemanden, der bei klarem Verstand
Aussagen macht, von denen er weiß, dass sie falsch sind.
..... mit sehr seltenen Ausnahmen

Worum könnte man Sie denn Ihrer Meinung nach beneiden?
It is loading more messages.
0 new messages